CHUYÊN ĐỀ HSG PHẦN GIỚI HẠN 11

You might also like

Download as pdf or txt
Download as pdf or txt
You are on page 1of 79

CHUYÊN ĐỀ GIỚI HẠN CỦA DÃY SỐ BỒI DƯỠNG HỌC SINH GIỎI

3.1. TÍNH GIỚI HẠN BẰNG ĐỊNH NGHĨA.

 1
a1  a  a

Bài 1. Cho dãy số  an  xác định bởi :  . Chứng minh rằng với mọi số thực
an 1  2an  2an  2
3 2

 3an 2  4an  1
a  0 thì dãy  an  hội tụ. Tùy theo a , hãy tìm giới hạn của dãy  an  .

Hướng dẫn giải


1
Nếu a  0 thì a   2 (do bất đẳng thức AM-GM).
a
1 1
Nếu a  0 thì a   2 (do bất đẳng thức AM-GM) nên a   2 .
a a

Nếu a  1 thì a1  2 . Ta chứng minh: an  2, n  *


.

Hiển nhiên a1  2 .

2.23  2.22  2
Giả sử ak  2  ak 1   2.
3.22  4.2  1
Vậy lim an  lim 2  2 .

a  0
. Nếu  thì a1  2 . Ta chứng minh an  2 n  *
.
a  1
Rõ ràng a1  2 . .

Giả sử ak  2 . Ta chứng minh ak 1  2 .

2ak 3  2ak 2  2
ak 1  2   2  2ak  ak  2   0 ( đúng).
2

3ak  4ak  1
2

Ta chứng minh  an  là dãy giảm, thật vậy :.

an3  2an 2  an  2   an  1  an  2 
2

n, an1  an    0.
3an 2  4an  1 3an 2  4an  1

( do tử âm, mẫu dương vì.

 2 7
 an 
3
3an 2  4an  1  0   .
 2 7
 an 
 3

2 7
Mà an  2   3an 2  4an  1  0 ).
3

Trang1
 an  giảm và bị chặn dưới   an  có giới hạn là L .

2an3  2an 2  2 2 L3  2 L2  2
lim an1  lim 
3an 2  4an  1 3L2  4 L  1 .
 L  2  an  2  L  1 

Vậy lim an  2 .

. Nếu a  0 thì a1  2 . Tương tự, ta có:.

an3  2an 2  an  2   an  1  an  2 
2

n, an1  an    0.
3an 2  4an  1 3an 2  4an  1

nên  an  tăng. Hơn nữa  an  bị chặn trên bởi 1 , thật vậy.

2ak 3  2ak 2  2
ak 1  1  1  ak  1 (2a  3)  0 .
2

3ak  4ak  1
2

Vậy  an  tăng và bị chặn trên   an  có giới hạn là L .

an  1, n , an 1  an  0, n
2 L3  2 L2  2 .
L  L  1  an  1 L  2 
3L  4 L  1
2

Vậy lim an  1 .

Tóm lại: + Nếu a  1 thì lim an  2 .

a  0
+ Nếu  thì lim an  2 .
a  1
+ Nếu a  0 thì lim an  1 .

 x1  0

Bài 2. Cho dãy số  xn  được xác định bởi 
n  
1 2 3 2015 . Tìm giới hạn
 xn 1  xn  x  x 2  x3   *

 n n n xn2015
của dãy nxn khi n   , với  là số thực cho trước.

Hướng dẫn giải


Dễ dàng chứng minh được xn  0, n  1 bằng qui nạp.

Ta có.
2
1  1 1
xn 1  xn  , n  1 xn21   xn    xn2  2  2  xn2  2 ; n  1 .
xn  xn  xn

Bởi vậy n  , n  2 thì xn2  xn21  2  xn22  4    x12  2  n  1 .

 xn  1, n  2 và lim xn   .
n 

Trang2
1 2 3 2015
Với n  *
, đặt xn1  xn   tn trong đó tn  2  3  2015 .
xn xn xn xn

t
xn  1; n  2  0  tn  , với t  2  3  2014  2015 (1), suy ra.
xn2
2
 1  1 2t
x2
n 1  x   xn   tn   xn2  2  tn2  2  2 xntn  n  2 .khi n   .
2
n
 xn  xn xn

 b1  x12
Áp dụng định lý trung bình Cesaro cho dãy  bn  với  .
bn  xn  xn 1 , n  2.
2 2

b1  b2  bn
ta có lim bn  2 suy ra lim  lim bn  2. .
n  n  n n

xn2  xn  xn1    xn1  xn2    x2  x1   x1 b1  b2  bn


2 2 2 2 2 2 2
n 1
Mà   suy ra lim 2  . .
n n n n  x 2
n

n 1
Thật vậy ta có thể chứng minh trực tiếp lim  như sau (chứng minh định lý trung bình Cesaro).
n  x 2 2
n

Xét dãy  cn  : c1  x12  2; cn  xn2  xn21  2 với n  2, 3 .


lim cn  0 nên   0 tồn tại m  *
sao cho cn  ,  n  m. .
n  2

Gọi M  max  ci  với 1  i  m 1 .


 2  m  1 M  2  m  1 M  m  1 M   .
Với  ở trên tồn tại m     1 thì  m ' hay
    m 2

Xét n  max m, m ' .ta có.


|  i 1ci |  im ci 
m 1
 n  m  1
2   m  1 M     m  1 M       . o đó theo định
n n
| ci |
  i 1

n n n n n 2 m 2 2
|  i 1ci |
n

nghĩa lim  0.
n  n

xn2  xn  xn1    xn1  xn2    x2  x1   x1 c1  c2  cn


2 2 2 2 2 2 2
n 1
   2 . suy ra lim 2  . .
n n n n  x 2
n

1
Nếu   2 thì n.xn  n.xn2  khi n   .
2

Nếu   2 thì n.xn  xn 2 .n.xn2   khi n   .

Nếu   2 thì n.xn  xn 2 .n.xn2  0 khi n   .

Trang3
Cho hai số a1 , b1 với 0  b1  a  1 .Lập hai dãy số  an  ,  bn  với n  1, 2,.. .Theo quy tắc
Bài 3. 1

1
sau: giải nghĩa cái đó là:. an 1  (an  bn ) , bn 1  an 1.bn Tính: lim an và lim bn .
2 . n n 

Hướng dẫn giải



Tính a2 , b2 với 0  b1  a1  1 ta có thể chọn 0  a  sao cho: b1  cosa ,.
2

Suy ra a1  cos 2 a .

1 1 a
a2  (cos 2 a  cos a)  cos a(cos a  1)  cos a.cos 2 .
2 2 2

a a
b2  cos a.cos 2 .cos a  cos a.cos .
2 2
Bằng quy nạp, chứng minh được:.
a a a a a
an  cos a.cos ...cos n 1 cos n 1 (1) bn  cos a.cos ...cos n 1 (2) .
2 2 2 2 2
a
Nhân hai vế của (1) và (2) cho sin và áp dụng công thức sin 2a được:.
2 n 1
a
sin 2a.cos
an  2n 1 , bn 
sin 2a
.
a a
2n.sin n 1 2n.sin n 1
2 2
Tính giới hạn:.
sin 2a sin 2a
lim an  , lim bn  .
n  2a n  2a
1 a
Bài 4. Cho dãy số  an  , a1  1 và an1  an  .Chứng minh: lim n  2 .
an n  n

Hướng dẫn giải


n n 1 n 1
1 1
a2
k 1  a  2  2   ai   a j   2  2(n  1). .
2
k
2 2

ak i 2 j 1 j 1 a j

n 1
1
a  2n  1  
2
n 2
. Vậy an  2n  1 , n  2. .
j 1 a j

1 1 1 1 1 1 1
ak2  2k  1 k  2        .
4
a k (2k-1) 2
(2k-1)  1 4k(k+1) 4  k  1 k 
2

n 1 n 1
1 1 1 1 1 1 5
Suyra:   (1  )     1  .
4
k  2 ak 4 n 1 4 4
j 1 a j 4 4

Trang4
n 1 n 1
1 1 5
Suyra:  2
 ( n  1)  4
 (n  1) (n  2). .
j 1 a j j 1 a j 4

5(n  1)
Vậy: an2  2n  1  (n  2) .
2

5(n-1) 1 a 5(n-1)
Suyra: n  2; 2n-1<a n < 2n-1+  2- < n  2n-1+ .
2 n n 2
an
Dođó: lim 2.
n  n
 
Bài 5. Cho hai số a1 , b1 với a1  cos 2 , b1  cos . Lập hai dãy số  an  ,  bn  với n  1, 2,... theo quy
8 8
1
tắc sau:. an 1  (an  bn ) , bn 1  an 1.bn . Tính: lim an và lim bn .
2 n n 

Hướng dẫn giải


+Tính a2 , b2 :.

1   1    
a2  (cos 2  cos )  cos (cos x  1)  cos .cos 2 .
2 8 8 2 8 8 8 16

    
b2  cos cos 2 cos  cos cos .
8 16 8 8 16
+ Bằng quy nạp, chứng minh được:.
      
an  cos cos 2
...cos n
cos n
(1) bn  cos cos 2
...cos (2) .
2.4 2 .4 2 .4 2 .4 2.4 2 .4 2 n .4

+Nhân hai vế của (1) và (2) cho sin và áp dụng công thức sin 2a được:.
2n .4
  
sin .cos n
sin
an  4 2 .4 , bn  4 .
n  n 
2 .sin 2 .sin
2n .4 2n .4
+Tính giới hạn:.
 
4sin 4sin
lim an  4 , lim bn  4.
n   n  
Bài 6. Cho dãy số  un  biết:.

 u1  1

 un , n  N * .
un 1  1  u 2
 n

Hãy tính lim (un n ) .


n

Trang5
Hướng dẫn giải
Ta có: u1  0  un  0 , n  N * .

un  1  un  un / (1  un2 )  un  (un3 ) / (1  un2 )  0 n  N * .

  un  là dãy số giảm và bị chặn dưới bởi 0 .

 lim un  a (a  R, a  0) .
n 

Từ un  1  un / (1  un2 ), cho n   ta được:.

a  a / (1  a 3 )  a  0. Vậy lim un  0 .
x 

Đặt vn  1/ (un2  1) 1/ (un2 ), n  N * .

Ta có vn  ((1  un2 ) / un )2 1/ (un2 )  2  un2  2 khi n   ? Áp dụng định lí trung bình Cesaro ta có:.

1 1

v1  v2  vn u 2
u12
lim  2  lim n 1
2.
n  n n  n

 1 1  1 1
 2  2  2  2
 lim  n 1
u u n  u n u1
 2.
n  n
1 1 1
2

u u n2 v u2 1
Mà lim n 1
 lim n  0 ; lim 1  lim  0 .
n  n n  n n  n n  n

1
u2 1 1
 lim n  2  lim 2
 2  lim (un n )  .
n  n n  n.u n  2
n

U1  2

Bài 7. Cho dãy U n  xác định bởi:  U n2  2009U n  n  N  .
*

U n 1 
 2010

 n
Ui 
Ta lập dãy Sn  với  S n    .Tính lim Sn .
i 1 U i 1  1 
x 

Hướng dẫn giải
a0
Tacó a1    0.
2

Giả sử a1 , a2 ,..., an 1  0 .

Tacó.

Trang6
 an an 1 a
   ...  0  0
1 2 n 1 1 1  1 1 1 1 
  an     an 1     an  2  ...     a0 .
 a a a  1 2   2 3   n n  1 
n 1
 n  2
 ...   0
0

 1 2 n
an 1 an  2 a1 a0
Hay an    ...   .
1.2 2.3 (n  1) n n( n  1)

Do a1 , a2 ,..., an 1  0 nên.

 an 1 an  2 a1   2an 1 3an  2 na 
   ...     ...  1 
 1.2 2.3 (n  1)n   1 2 n 1 
2
.
a a a1  a02
  n 1  n  2  ...   
 1 2 (n  1)  n2

 an 1 an 2 a1  a02
   ...   .
 1.2 2.3 (n  1)n  2  2an 1 3an2 na1 
n    ...  
 1 2 n 1 
Ta lại có.
2an 1 3an  2 na  2a 3a a 
  ...  1  n  n 1  n 2  ...  1 
1 2 n 1  n 2n n 1 
.
a a a   a 
 n  n 1  n  2  ...  1   n   0    a0 .
 1 2 n 1   n

a a a1  a0
  n1  n2  ...   2 .
 1.2 2.3 (n  1)n  n

an 1 an 2 a1 a0 a a0
 an    ...     02  0.
1.2 2.3 (n  1)n n(n  1) n n(n  1)
Từ đó suy ra điều phải chứng minh.

1  un2  1
Bài 8. Cho dãy số  un  xác định bởi u1  1, un1  , n  1.
un

a) Chứng minh:.

un  tan , n  1. .
2n 1
b) Suy ra tính đơn điệu và bị chặn của  un  .

HƯỚNG DẪN GIẢI


a) Chứng minh bằng quy nạp toán học.
   
b) Nhận xét 0  n 1
 , n  1 và hàm số tanx đồng biến trên  0;  .
2 4  4

nên dãy số  un  giảm và bị chặn dưới bởi số tan 0  0 .

Trang7

và bị chặn trên bởi số tan  1.
4 .

Bài 9. Cho dãy số  xn  xác định bởi:.

1 2 3 2014 2015
x1  0; xn1  xn   2  3  ...  2014  2015 , n  *
..
xn xn xn xn xn

n
1.Với mỗi n  *
,đặt yn  .Chứng minh dãy số  yn  có giới hạn hữu hạn và tính giới hạn đó.
xn2

2.Tìm các số  để dãy  nxn  có giới hạn hữu hạn và giới hạn là một số khác 0 .

HƯỚNG DẪN GIẢI


1 1
1.Từ giả thiết suy ra xn1  xn   0  xn21  xn2  2  2  xn2  2
xn xn
.
Suy ra xn21  xn2  2  xn21  2  ...  x12  2n do đó lim xn   .

Xét
 1 2 3 2014 2015  1 2 3 2014 2015 
xn21  xn2   xn 1  xn  xn 1  xn    2 xn   2  3  ...  2014  2015   2  3  ...  2014  2015 
 xn xn xn xn xn  xn xn xn xn xn 

.
 1 2 3 2014 2015  2 3 2014 2015 
  2  2  3  4  ...  2015  2016 1   2  ...  2013  2014 
 xn xn xn xn xn  xn xn xn xn 
.

Suy ra lim  xn21  xn2   2


.

xn2  xn  xn1    xn1  xn2   ...   x2  x1   x1


2 2 2 2 2 2 2

Ta có  .
n n
Áp dụng định lý trung bình Cesaro ta có.

xn2
lim  lim
 xn2  xn21    xn21  xn22   ...   x22  x12   x12
2 .
n n
n 1
Do đó lim 
xn2 2
.
n  2
2.Xét zn  nxn  xn
xn2
.
Từ đó:.
+) Nếu   2 thì lim zn  

Trang8
+)Nếu   2 thì lim zn  0

.
1
+) Nếu   2 thì lim zn 
2
.
Vậy   2 là giá trị cần tìm thỏa mãn đề bài.

Bài 10. Cho dãy số  yn  thỏa mãn y1  0, yn31  y1  y2  ...  yn , n  1 .

y 
Chứng minh rằng dãy số  n  có giới hạn bằng 0 khi n   .
n
Hướng dẫn giải

Từ giả thiết ta có yn31  yn  yn3 , n  2 , do đó dãy số  yn n 2 là dãy tăng, vì.

vậy yn31  yn  yn3  yn ( yn2  1)  yn1 ( yn2  1) .

 yn21  yn2  1 , n  2  yn21  yn2  1  ...  y22  n  1 .

y22  n  1 y22  n  1
2
 yn 1 
   . Mà lim  0 nên theo định lý kẹp ta có.
 n 1  (n  1)2 (n  1) 2
2
 y  y y
lim  n1   0  lim n 1  0  lim n  0 .
 n 1  n 1 n

un  (0;1)
Bài 11. Tìm tất cả các hằng số c  0 sao cho mọi dãy số dãy số (un ) thỏa mãn:  n  1 .
un 1 (1  un )  c

đều hội tụ. Với giá trị c tìm được hãy tính giới hạn của dãy (un ) .

Hướng dẫn giải


Ta xét các trường hợp sau.
1 c cun
+ Nếu c  , thì từ giả thiết, ta có un1    4cun ; n  1 .
4 1  un un (1  un )

1
Từ đây bằng quy nạp, ta suy ra un  (4c)n1 u1 . Do 4c  1 nên un   khi n   . Do đó, c 
4
không thỏa mãn.

1  1  1  4c 1  1  4c  a(1  b)  c
+ Nếu 0  c  , thì tồn tại a, b   ;  , a  b sao cho  . Thật vây, lấy
4  2 2  b(1  a)  c
 1  1  4c 1  1  4c 
a   ;  , đặt b  a  x ( x  0) , thì.
 2 2 
a (1  a )  c
a (1  b)  c  a (1  a  x)  c  x  .
a

Trang9
Chú ý là b(1  a )  a (1  a )  c. Do đó, ta chỉ cần chọn x  0 như trên và b  a  x, thì được 2 bất đẳng
thức nêu trên.
Xét dãy số (un ) xác định bởi.

a nêu n  2m
un   .
b nêu n  2m  1
1
thì dãy (un ) thỏa mãn giả thiết nhưng không hội tụ. Thành thử, 0  c  cũng không thỏa mãn.
4
1 1 un
+ Nếu c  , thì un1    un . Suy ra dãy (un ) tăng và bị chặn. Do đó, (un ) hội tụ.
4 4(1  un ) 4un (1  un )

1 1 1
Đặt x  lim un , thì từ giả thiết ta có x(1  x)  hay x  . Vậy lim un  . .
4 2 2

 1
 x1 
 2
Bài 12. Cho dãy số (xn) thỏa mãn:  2
. Chứng minh dãy số trên có giới hạn.
 x  x  xn ; n  1


n 1 n
n2

Hướng dẫn giải


n  n  1
*) Ta chứng minh xn  n2  với mọi n  1 (1).
2
Thật vậy: n  1 đúng.
k  k  1
Giả sử (1) đúng với n  k  1: xk  k 2  .
2

xk2
  k  1  k2  xk  k 2    k  1 .
x
 xk 1   k  1  xk 
2 2 2
2
k k

 k  1  k  k  1 3  k  1 k  k  1
2

  1   k  1  
2
.
k 2  2 2 2

k  1  3  k  1   k  1 k  2 
  k  (đpcm).
2  2  2

*) Ta chứng minh  xn  có giới hạn.

NX:  xn  tăng và xn  0 với mọi n .

1 1 1 2 1 1  1 1
Ta có       2 1    2  xn  với mọi n  1.
xn xn 1 xn  n 2
n  n  1 x1 xn  n 2 2

Vậy  xn  có giới hạn.

Trang10
un4  20132
Bài 13. Cho dãy số  un  xác định bởi u1  2014, un1  , n  *
. Đặt
un3  un  4026
n
1
vn   , n  *
. Tính lim vn .
k 1 u  2013
3
k

Hướng dẫn giải

un4  20132 (un  2013)(un3  2013)


+ Ta có un 1  2013   2013  (1).
un3  un  4026 (un3  2013)  (un  2013)

Từ đó bằng quy nạp ta chứng minh được un  2013, n  *


.

1 1 1 1 1 1
+ Từ (1) suy ra   3  3   .
un1  2013 un  2013 un  2013 un  2013 un  2013 un1  2013
n
 1 1  1 1 1
Do đó vn        1 .
k 1  uk  2013 uk 1  2013  u1  2013 uk 1  2013 uk 1  2013

+ Ta chứng minh lim un   .

un2  4026un  20132 (un  2013)2


Thật vậy, ta có un1  un   3  0, n  *
.
un3  un  4026 un  un  4026

Suy ra  un  là dãy tăng, ta có 2014  u1  u2  ... .

a 4  20132
Giả sử  un  bị chặn trên và lim un  a thì a  2014 . Khi đó a  .
a 3  a  4026

 a  2013  2014 ( vô lí). Suy ra  un  không bị chặn trên, do đó lim un   .

1
Vậy lim vn  lim (1  ) 1.
uk 1  2013

u1  2013 un21


Bài 14. Cho dãy số  un  xác định bởi:  . Tìm lim .
un 1  un  2, n  n u 2 .u 2 ...u 2
2 *
1 2 n

Hướng dẫn giải


1
- Vì u1  2013  2 nên đặt u1  a  , a > 1.
a
2
 1 1
Ta có u2  u  2   a    2  a 2  2 .
2
1
 a a
Bằng quy nạp, ta chứng minh được.
1
un1  a 2  , n 
n

n .
a2
- Xét.

Trang11
1 1
n n
 i1 1   1   1  n  2i1 1   1   2n 1 

i 1
ui    a 2  2i1
i 1  a

 
 
a   
a  
a     a  2i1
a  i 1  a
    a  a   a  2n  1.0
    a 
2
 1  2n 1 
a    a  2n  2 2
.
un21 a   lim un 1   a  1    a  1   4  20132  4 1.0
2
 2 2 2 
a 
   
 a  a
2 n  u 2 .u 2 ...u 2
u1 .u2 ...un  2n 1 
 2n 
1 2 n
a
 a 
Bài 15. Cho dãy số (an ) thỏa mãn: lim(5an1  3an )  4 . Tính lim an .

Hướng dẫn giải


Đặt an  2  bn . Từ giả thiết suy ra lim (5bn 1  3bn )  0 .

Với số dương  bé tùy ý, tồn tại số N sao cho với n  N thì ta có:.

5bn 1  3bn  (1).
5

- Nếu bn 1.bn  0 thì từ (1) dẫn đến 5bn 1  3bn   bn   .
5
- Xét trường hợp bn 1.bn  0 hay bn 1 , bn cùng dấu, chẳng hạn chúng cùng dương.

 
. Nếu 2bn 1  bn  0 thì kết hợp với (1): 3(2bn 1  bn )  bn 1  dẫn đến bn 1  .
5 5

Mà từ (1) ta có 3bn  5bn 1   bn   .
5

5 1 
. Nếu 2bn 1  bn  0 thì kết hợp với (1): (bn1  bn )  bn  dẫn đến bn   .
2 2 5

Tóm lại luôn có bn   , hay lim(bn )  0 .

Vậy lim(an )  2 .

un2015  2un  4
Bài 16. Cho dãy (un ) xác định như sau: u1 = 3 và un1  , n  1, 2, 3... . Với mỗi số
un2014  un  6
n
1
nguyên dương n , đặt vn   . Tìm lim vn .
i 1 u
i
2014
4 n 

Hướng dẫn giải

un2015  2un  4 (un  2)(un  4)


Đặt   2014 ta có un1  2   2  , (*) .
un2014  un  6 (un  4)  (un  2)

Bằng quy nạp ta chứng minh được un  3, n  1 .

un 1  2un  4 (un  2)2


Xét un1  un    un    0, un  3 .
un  un  6 un  un  6
Do đó (un ) là dãy tăng và 3  u1  u2   un  .

Trang12
a 1  a  4
Giả sử (un ) bị chặn trên, suy ra lim un  a , a  3 . Khi đó ta có a   a  2  3 (vô lí), suy ra
n  a  a  6
(un ) không bị chặn trên. Vậy lim un   .
n 

1 1 1 1 1 1
Từ (*) suy ra    , hay    .
un1  2 un  2 un  4 un  4 un  2 un 1  2
n
1 n
 1 1  1
vn        1
ui  4 i 1  ui  2 ui 1  2 
 .
i 1
2014
un1  2
1
Vậy lim vn  lim (1  ) 1.
n  n un1  2

u1  3

Bài 17. Cho dãy số  un  được xác định bởi  3 . Chứng minh rằng dãy  un 

u n 1  3u n 1  2  u n , n  1
có giới hạn hữu hạn và tìm giới hạn đó.

Hướng dẫn giải

u1  3

Dãy số  un  được xác định bởi  3 .

u n 1  3u n 1  2  u n , n  1

Ta chứng minh un  2, n  1 .

Thật vậy ta có u1  3  2 .

Giả sử uk  2, k  1 , khi đó uk31  3uk 1  2  uk  2  2  2 nên.

uk31  3uk 1  2  0   uk 1  1  uk 1  2   0  uk 1  2 .
2

Do đó theo nguyên lý quy nạp thì un  2, n  1 .

Xét hàm số f  t   t 3  3t trên khoảng  2,    .

Ta có f '  t   3t 2  3  0, t  2 .

Do đó hàm số f  t  đồng biến trên khoảng  2,    .

Mặt khác ta có u13  3u1  18  5  u23  3u2  f  u1   f  u2   u1  u2 .

Giả sử uk  uk 1  k  1  2  uk  2  uk 1  uk31  3uk 1  uk32  3uk 2 .

 f  uk 1   f  uk  2   uk 1  uk  2 .

Do đó un  un 1 , n  1  Dãy  un  là dãy giảm và bị chặn dưới bởi 2 nên dãy  un  có giới hạn hữu hạn.

Giả sử lim un  a  a  2  . Từ hệ thức truy hồi un31  3un 1  2  un chuyển qua giới hạn ta được:.

   2  a   a  2   a5  2a 4  2a3  4a 2  a  1  0 .
2
a3  3a  2  a  a3  3a

Trang13
 
  a  2 a 2  a3  4  2a3  a  1  a  1  0  a  2  a  2  .

Vậy lim un  2 .

 xn  thỏa   n  N  (*).


2
Bài 18. Cho dãy số mãn: x1  2015 và xn1  xn . xn  1 *

n
1
Tìm: lim  .
i 1 xi  1

Hướng dẫn giải

* Ta có: xn  0 n  N * .

xn1
 
xn  1  0 n  N *   xn  là dãy số tăng.
2
Và: 
xn

* Đặt un  xn .

 un xác định vì xn  0 n  N * và un  0 n  N * .

 un 1  xn 1  xn 1  un21 .

Nên từ giả thiết (*) ta có:.

un21  un2 .  un  1   un .  un  1  .
2 2

 un1  un2  un n  N * (1).

* Xét dãy số  un  ta có:.

. un1  un  un2  0 n  N *   un  tăng.

. Giả sử  un  có giới hạn là a . Từ (1) ta có:.

a  a 2  a  a  0 (loại).

  un  tăng và không bị chặn  lim un   .

* Ta có:.

1 un2 u u u u 1 1
  2n 1 n  n 1 n   .
un  1  un  1 un  un  un  .un
2
un1.un un un1

n
1 1 1
   .
i11 ui  1 u1 un 1
n
1 1 1  1
 lim   lim    .
i 1 ui  1  u1 un 1  2015
n
1 1
Vậy: lim   ..
i 1 xi  1 2015

Trang14
u1  5
Bài 19. Cho dãy số un  ; (n = 1; 2;.) được xác định bởi:  . Chứng minh dãy số un  có
un 1  un  12
giới hạn. Tìm giới hạn đó.

Hướng dẫn giải


Dự doán giới hạn của dãy số,bằng cách giải phương trình:.

a  0
a  a  12   2  a  4.
a  a  12
Nhận xét u1  5 .

u2  u1  12  17  u1 .

u3  u2  12  17  12  u2 ... .

Ta dự đoán dãy số un  là dãy số giảm và bị chặn dưới bởi 4 tức là un  4 .

Chứng minh dãy số un bị chặn: tức là un  4 .

khi n  1, u1  5  4 vậy n  1 đúng.

Giả sử uk  4 , ta chứng minh: uk 1  4 .

Thật vậy ta có:.

uk 1  uk  12  0  uk21  uk  12  uk21 12  uk  4  uk21  16  uk 1  4 .

Vậy dãy số un bị chặn dưới.

Ta chứng minh dãy số un  là dãy số giảm.

Ta có:.

un2  un  12 (un  4)(un  3)


un 1  un  un  12  un   un1  un   0 (vì un  4 ).
un  12  un un  12  un

Vậy dãy số un  giảm và bị chặn dưới nên có giới hạn.

Đặt lim un  a thì lim un 1  a. .

Ta có:.

un 1  un  12  lim un 1  lim un  12  a  a  12  a  4 .

Vậy lim un  4. .

Bài 20. Cho dãy số  xn  được xác định bởi.


 x1  2,1

 xn  2  xn2  8 xn  4 .
 n1
x  
* , n  1, 2,...
 2

Trang15
n
1
Với mỗi số nguyên dương n, đặt yn   . Tìm lim yn .
i 1 x 42
i

Hướng dẫn giải


Ta có kết quả sau: với số thực a  2 bất kì, ta có.

a2 a  8a  4 a2 a  4a  4 a  2  a  2
2 2

  a.
2 2 2

Do đó 2,1  x1  x2  ...   xn  là dãy tăng, giả sử bị chặn trên tức là có giới hạn lim xn  L  2 .

Chuyển qua giới hạn điều kiện (*) ta có phương trình.

x2 x  8x  4
2

x  x  4   x  3  x  2  .
2

phương trình này không có nghiệm hữu hạn lớn hơn 2.


Suy ra dãy  xn  tăng và không bị chặn trên nên lim xn   .

xn  2  xn  8 xn  4
2

Ta có xn 1   2 xn 1  xn  2  xn  8 xn  4 .
2

  2 xn 1  xn  2   xn  8 xn  4  xn  2  4   xn  3  xn  2  .
2 2 2

1 xn  3 xn  2  1 1 1
     .
xn  2 xn 1  4 xn 1  4 xn 1  2 xn 1  4
2 2 2

1 1 1
   .
xn 1  4 xn  2 xn 1  2
2

n
1 1 1 1
Suy ra yn      10  .
xi  4 x1  2 xn 1  2 xn 1  2
2
i 1

Vậy lim yn  10 .

Bài 21. Cho dãy số  xn  được xác định bởi x1  2016, xn1  xn2  xn  1, n  1, 2,3,... .

a)Chứng minh rằng  xn  tăng và lim xn   .

1 1 1
b)Với mỗi số nguyên dương n , đặt yn  2016    ...   . Tính lim yn . .
 x1 x2 xn 

Hướng dẫn giải

a)Ta có
xn1  xn  xn2  2 xn  1   xn  1  0  xn1  xn , n  1.
2

Do đó
 xn  tăng.
Ta chứng minh bằng quy nạp theo n rằng xn  n  1, n  1 (1).

Thật vậy, (1) đúng với n  1 .Giả sử (1) đúng với n ( n  1) thì.

xn1  xn  xn  1  1  n  n  1  1  n2  n  1  n  2 .

Trang16
Vậy (1) đúng với mọi n. Từ  xn  tăng ngặt và xn  n  1, n  1 suy ra lim xn  . .

1 1 1 1
b)Ta có xn 1  1  xn  xn  1 . Suy ra    .
xn 1  1 xn  xn  1 xn  1 xn

1 1 1
Từ đó  
xn xn  1 xn1  1 .

1 1 1  1 1   1 1 
 yn  2016    ...    2016     2016   
 x1 x2 xn   x1  1 xn 1  1   2015 xn 1  1  .

1 2016
Từ lim xn    lim  0 . Vậy lim yn  ..
xn 2015

1 1 1 a 
Cho dãy  an n 1 : an  sin1  22 sin  32 sin  ...  n 2 sin

Bài 22. n  1 . Chứng minh dãy  n2 
2 3 n  n  n 1
a
hội tụ và tính lim n2 .
n

Hướng dẫn giải


1
Bổ đề 1: x  sin x  x  x 3x  0 .
6
1 1 1
1    ... 
Bổ đề 2: lim 2 3 n  0.
n
1 1 1 1 1 1
Đặt xn  n 2 sin . Áp dụng bổ đề 1:  sin   3  k  xk  k  .
n k k k 6k 6k

1 1 1
 1  2  ...  n  an  1  2  ...  n  1   ...   .
6 2 n
1 1
1   ... 
1 a 1 2 n.
Chia các vế cho n 2 :  n2   2
2 n 2 6n
an 1
Cho n   , và lấy giới hạn, suy ra lim  .
n2 2 .

 n  1
2
un
Bài 23. Cho dãy số u1  2, un1  n  1 . Tính giới hạn lim .
un  1 n  n

Hướng dẫn giải


n2
Ta chứng minh quy nạp  un  n  1 , n  1 .
n 1
Rõ ràng khẳng định đã đúng với u1 .

Trang17
 k  1  u  k  2 .
2
k2
Giả sử đã có  uk  k  1, k  1 . Ta chứng minh k 1
k 1 k 2

(k  1)2  k  1
2

Thật vậy: uk  k  1  uk 1   .
uk  1 k 2

(k  1)2  k  1
2
k2 1
uk   uk 1   2  k 2 2  k  2 ..
k 1 uk  1 k k  k 1
1
k 1
n2 u
Vậy ta có  un  n  1, n  1  lim n  1 .
n 1 n  n

x 1  
Cho   2 và dãy số  x n  với: 

Bài 24. n 3 .
2x n 1  3x n 
2
n  N *

 n

a) Chứng minh: x n  1 với n  N * .

b)Chứng minh dãy số x n  có giới hạn và tìm giới hạn đó.

Hướng dẫn giải


Ta chứng minh x n  1 với n  N * bằng quy nạp.

Ta có: x1   nên x 1  1 .

Giả sử: x k  1 với k  N * .

n 1 n3
Ta có: 3x k  3 và  1 nên 3x 2n   2 . Suyra: x n 1  1 .
2

n n
Vậy x n  1 với n  N * .

Ta chứng minh  x n  là dãy giảmbằng quy nạp.

Vì   2 nên 3 2  4  2 .Ta có x 2  x 1 .
n 1
Giả sử: x k 1  x k . Ta có: 3 x k 1  3x k và f n  =
2 2
là hàm nghịch biến nên:.
n
k4 k3
3x 2k 1   3x 2k  .
k 1 k

Suy ra: x k 2  x k 1 . Vậy  x n  là dãy giảm.

xn  lả dãy giảm và bị chặn dưới bởi 1 nên hội tụ.


 x1  1

Đặt lim x n   .Ta có 2  3  1    1 .  xn  3xn  4 (n  N * )  un  un  x2 n 1  n  N * 
2
 .
 xn 1  x  1
 n

Vậy lim x n  1 .

Trang18
u1  2011

Bài 25. Cho dãy số  un  được xác định:  n .

 2 u n 1  2 n
u n  1 , n  N *

Chứng minh rằng dãy số  un  có giới hạn hữu hạn và tính giới hạn đó.

Hướng dẫn giải


1
Ta có 2 n un 1  2 n.un  1  un1  un  .
2n

Chứng minh : un  21– n (bằng quy nạp).

*với n  1 ta có u1  2011  20 .

*Giả sử uk  21– k (với k  1 ).

*Cần chứng minh : uk 1  2– k .

Ta có uk 1  uk  2  k  21k  2  k  2  k . Suy ra điều phải chứng minh.

1
Từ đó ta có un – 2– n  0 với mọi n  un 1  un  .
2n
1 1 1 1
Ta có u2  u1  ; u3  u2  2 ; u4  u3  3 ;...; un  un 1  n 1 .
2 2 2 2

1 1 1 1 
 un  u1    2  3  ...  n 1  .
2 2 2 2 
n 1
1
1   n 1

Công thức tổng quát : un  2011 .  


1 2 1
 2011 1    .
2 1 2
2
Vậy lim un  2010.

u1  a

Bài 26. Cho số thực a   0;1 , xét dãy số  un  với:  1 2013 .
un 1  un2  un , n   
 2014 2014

a) Chứng minh rằng: 0  un  1, n   .

b) Chứng minh rằng  un  có giới hạn hữu hạn. Tìm giới hạn đó.

Hướng dẫn giải

a) Chứng minh: 0  un  1, n   1 .

n  1: u1  a   0;1  1 đúng với n=1.

1 1
Giả sử 0  uk  1 với k  1, k   . Ta có: 0  uk2  1  0  uk2  .
2014 2014

Trang19
2013 2013
0  uk  1  0  uk  .
2014 2014
1 2013
0 uk2  uk  1  0  uk 1  1 .
2014 2014

Vậy: 0  un  1, n   .

b) Chứng minh rằng  un  có giới hạn hữu hạn. Tìm giới hạn đó.

Ta chứng minh:  un  là dãy tăng.

n    , un 1  un 
1
2014
un2 
2013
2014
un  un 
1 
2014 

un  un u n 
 un  2013   0 .

 un1  un , n  hay  un  là dãy tăng.(2).

Từ (1),(2) suy ra  un  có giới hạn hữu hạn.Giả sử  un  có giới hạn là a,  o  a  1 .

1 2013
Ta có: a  a2  a  a  1 . Vậy lim un  1 .
2014 2014

 3
u1  2
Bài 27. Cho dãy số(un) xác định như sau:  .
1 2
u  u  , n  N
3 
 n 1 3 n 3

a) Chứng minh rằng: 1  un  2, n   .

b) Chứng minh rằng  un  có giới hạn hữu hạn. Tìm giới hạn đó.

Hướng dẫn giải


3
a) Với: n  1: u1   1 đúng với n=1.
2
Giả sử: 1  uk  2 với k  1, k   .

Ta có: uk 1  2  uk3    uk  2   uk2  2uk  4   0  uk 1  2 .


1 8 1
3 3 3

uk 1  1 
3
 uk  1  0  uk 1  1 .
1 3

 1  uk 1  2 . Vậy: 1  un  2, n   .

1
b) n    , un 1  un   un  1  un  2   0  un1  un , n  hay  un  là dãy giảm (2).
2

Từ (1),(2) suy ra  un  có giới hạn hữu hạn.

Gọi a là giới hạn của  un  , 1  a  2 .

1 2
Ta có a  a 3   a  1 . Vậy lim un  1 .
3 3
Trang20
un2
Bài 28. Cho dãy số  un  xác định bởi: u1  1; un1   un , n  N * . Tìm giới hạn sau:
2015
u u u 
lim  1  2  ...  n  .
n  u
 2 u3 un 1 

Hướng dẫn giải

un2 u 1 1 
Từ đề bài ta có: un 1  un  . Suy ra: n  2015   .
2015 un 1  un un 1 

u1 u2 u 1 1   1 
Ta có:   ...  k  2015     2015 1   .
u2 u3 uk 1  u1 uk 1   uk 1 

Ta có  un  là dãy đơn điệu tăng và u1  1 .

2
Nếu lim un   thì       0 .
n 2015

( vô lí vì  un  là dãy đơn điệu tăng và u1  1 ).

Suy ra: lim un   .


n 

u u u 
Kết luận: lim  1  2  ...  n   2015 .
n  u
 2 u3 un 1 

u1  2013
Bài 29. Cho dãy số  un  xác định bởi  2  n  N *  . Chứng minh rằng dãy (un) có
un  2un .un 1  2013  0
giới hạn và tính giới hạn đó.

Hướng dẫn giải


Từ hệ thức truy hồi suy ra 2un .un1  un 2  2013 .

Bằng quy nạp chứng minh được un > 0, với mọi n.

Do đó ta có:.

un 12  2013 1  2013  2013


un    un 1    un .  2013, n  1 .
2un 1 2 un 1  un

Mặt khác ta có :.

un1 un 2  2013 1 2013 1 1


      1.
un 2un 2 2 2un 2 2 2

(un) là dãy số giảm và bị chặn dưới bởi 2013 , do đó (un) có giới hạn hữu hạn.

Đặt lim un  a .

a 2  2013
Ta có : a   a  2013 . Vậy lim un  2013 .
2a

Trang21
xn4  9
Bài 30. Cho dãy số  xn  xác định bởi: x1  4, xn1  , n  *
.
xn3  xn  6

a) Chứng minh rằng lim xn   ;.


n 

n
1
b) Với mỗi số nguyên dương n , đặt yn   . Tính lim yn .
k 1 x 3
3
k

Hướng dẫn giải

xn4  9  xn  3  xn3  3
a) Xét xn 1  3  3  * .
xn  xn  6  xn3  3   xn  3

Bằng quy nạp chứng minh được xn  3, n  1 .

xn4  9 xn2  6 xn  9
Xét xn1  xn   x  .
xn3  xn  6 xn3  xn  6
n

 xn  3
2

 xn1  xn   0, n  *
.
xn3  xn  6

Do đó  xn  là dãy tăng và 4  x1  x2  x3  ... .

Giả sử  xn  bị chặn trên  lim xn  a .

a4  9
Do đó: a   a  3  4 (vô lý). Suy ra  xn  không bị chặn trên. Vậy lim xn   .
a3  a  6
1 1 1 1 1 1
b) Từ (*), suy ra:   3  3   .
xn1  3 xn  3 xn  3 xn  3 xn  3 xn1  3
n
1 n
 1 1  1
Suy ra: yn         1 .
k 1 xk  3 k 1  xk  3 xk 1  3 
3
xn 1  3

 1 
Vậy lim yn  lim 1    1.
 xn 1  3 

 x1  1
 x12014 x22014 xn2014
Bài 31. Cho dãy số  2015
xn . Tìm giới hạn của dãy số u n với u    ...  .
 
n
 n 1
x x x2 x3 x n 1

n
2015

Hướng dẫn giải


xn2015 x 2015 x x xn2015
xn1   xn  xn1  xn  n  n1 n 
2015 2015 xn1 xn 2015xn1 xn
.

Trang22
1 1 xn2014 1 1  xn2014
    2015   
xn xn 1 2015 xn 1  n
x xn 1  xn 1

 1 
Từ đó un  2015 1  .
 xn 1 

Dễ thấy  xn  là dãy tăng và 1  x1  x2  x3  ... .

Giả sử  xn  bị chặn trên  lim xn  a .

a 2015
Do đó: a   a  a  0  1 (vô lý). Suy ra  xn  không bị chặn trên. Vậy lim xn   .
2015

 1 
Vậy limu n  lim 2015 1    2015 .
 xn 1 

 x1  1

Bài 32. Cho dãy số {xn } xác định bởi  xn2 . Tìm giới hạn của dãy ( Sn ) với
 n 1
x  x 

n
2015
x x x
Sn  1  2  ...  n .
x2 x3 xn1

Hướng dẫn giải

xn2 x x x2 x 1 1 
xn 1  xn   2015  xn 1  xn   xn2  2015 n 1 n  n  n  2015   . .
2015 xn 1 xn xn 1 xn xn 1  n
x xn 1 

x1 x2 x 1 1   1 
Suy ra: Sn    ...  n  2015     2015 1  .
x2 x3 xn 1  x1 xn 1   xn 1 

Dễ thấy  xn  là dãy tăng và 1  x1  x2  x3  ... .

Giả sử  xn  bị chặn trên  lim xn  a .

a2
Do đó: a   a  a  0  1 (vô lý). Suy ra  xn  không bị chặn trên. Vậy lim xn   .
2015

 1 
Vậy limSn  lim 2015 1    2015 .
 xn 1 

 x1  1
 n
1
Bài 33. Cho dãy số ( xn ) xác định bởi  . Đặt Sn   .
 xn 1  xn ( xn  1)( xn  2)( xn  3)  1
 k 1 xk  2

Tìm limSn .

Hướng dẫn giải

xn 1  xn ( xn  1)( xn  2)( xn  3)  1  ( xn 2  3xn )( xn 2  3xn  2)  1  xn2  3xn  1


.

Trang23
n
1 1 1 1 1 1 1 1
Ta có    Sn       .
xn  2 xn  1 xn1  1 k 1 xk  2 x1  1 xn 1  1 2 xn 1  1

Dễ thấy: xn1  xn   xn  1  0, n  N * suy ra  xn  là dãy tăng và 1  x1  x2  x3  ... .


2

Giả sử  xn  bị chặn trên  lim xn  a .

Do đó: a  a 2  3a  1  a  1  1 (vô lý). Suy ra  xn  không bị chặn trên. Vậy lim xn   .

1 1  1
Vậy limSn  lim    .
 2 xn 1  1  2

 2016
u1  1 1 1
Bài 34. Cho dãy số (un) xác định bởi:  2015 . Đặt Sn   . . .
2u  u 2  2u , n  * u1  2 u2  2 un  2
 n 1 n n

Tính: limSn.

Hướng dẫn giải


un  un  2  1 1 1 1 1 1
2un 1  un  un  2   un 1       
2 un1 un un  2 un  2 un un 1 .

n
1 1 1 2015 1
 Sn       .
k 1 uk  2 u1 un 1 2016 un 1

Bằng quy nạp ta dễ dàng chứng minh được un  0, n  N * .

Khi đó: un 1  un  un 2  0, n  N * suy ra  un  là dãy tăng và


1 2016
 u1  u2  u3  ... .
2 2015

Giả sử  un  bị chặn trên  limu n  a .

2016
Do đó: 2a  a 2  2a  a  0  (vô lý). Suy ra  un  không bị chặn trên.
2015
Vậy limu n   .

 2015 1  2015
Vậy limSn  lim    .
 2016 un 1  2016

xn4  9
Bài 35. Cho dãy số  xn  xác định bởi: x1  4, xn1  , n  *
.
xn3  xn  6

a) Chứng minh rằng lim xn   ;.


n 

n
1
b) Với mỗi số nguyên dương n , đặt yn   . Tính lim yn .
k 1 x 3
3
k

Hướng dẫn giải

Trang24
xn4  9  xn  3  xn3  3
a) Xét xn 1  3  3  * .
xn  xn  6  xn3  3   xn  3

Bằng quy nạp chứng minh được xn  3, n  1 .

xn4  9 xn2  6 xn  9
Xét xn1  xn   x  .
xn3  xn  6 xn3  xn  6
n

 xn  3
2

 xn1  xn   0, n  *
.
x  xn  6
3
n

Do đó  xn  là dãy tăng và 4  x1  x2  x3  ... .

Giả sử  xn  bị chặn trên  lim xn  a .

a4  9
Do đó: a   a  3  4 (vô lý). Suy ra  xn  không bị chặn trên. Vậy lim xn   .
a3  a  6
1 1 1 1 1 1
b) Từ (*), suy ra:   3  3   .
xn1  3 xn  3 xn  3 xn  3 xn  3 xn1  3
n
1 n
 1 1  1
Suy ra: yn         1 .
k 1 xk  3 k 1  xk  3 xk 1  3 
3
xn 1  3

 1 
Vậy lim yn  lim 1    1.
 xn 1  3 
3.2. TÍNH GIỚI HẠN BẰNG CÁC CÔNG THỨC CƠ BẢN

u1  1

Bài 1. Cho dãy số  an  thỏa mãn  1 . Tìm tất cả các số thực a sao cho dãy số

un1  un 
3 u
 n  *
 n

ua
 xn  xác định bởi xn  n ( n * ) hội tụ và giới hạn của nó khác 0.
n
Hướng dẫn giải

Từ giả thiết ta có dãy số  un  là dãy số dương và tăng(1).

1
Giả sử  un  bị chặn trên suy ra nó hội tụ. Đặt L  lim un , ta có ngay L  L  3
(vô lý).
L

Vì vậy  un  không bị chặn trên (2).

Từ (1) và (2) ta có lim un   .

 4 4
 1
Xét lim  un31  un3  . Đặt vn  4 ( n * ), ta có lim vn  0 .
  un3

Trang25
4
 1 
3 4
4 4
un31  un3   3  vn4   
1 1 1  vn  3 1

vn3  4vn2  6vn  4
.
  v 8 4
 v4
 n


n vn
1  vn   1  vn   1
3 3

4
 4 4
 4 u3 4
Suy ra lim  un31  un3   . Từ đó lim n  (sử dụng trung bình Cesaro).
  3 n 3

 4
 khi a 
3
 34
 
una  un a  43   4
Ta có lim  lim  0 khi a  .
 n n  
.u
n    3
  4 4
3 khi a 
 3
4
Vậy a  là giá trị cần tìm.
3

 1
u1  2 ; u2  3
Bài 2. Cho dãy số  un  xác định như sau: 
u .u  1
un  2  n 1 n , n  N *
 un 1  un
a) Chứng minh rằng tồn tại vô số giá trị nguyên dương của n để un  1 .

b) Chứng minh rằng  un  có giới hạn hữu hạn, tìm giới hạn đó.

Hướng dẫn giải


 u  1 un  1 (1).
a) Trước hết ta luôn có un  0, n  N * . Xét un 2  1  n1
un1  un

Bằng phương pháp quy nạp ta chứng minh được u3n , u3n1  1, n  N * và u3n  2  1, n  N * .

Từ đó suy ra điều phải chứng minh.


 u  1 un  1 (2).
b) Ta có un 2  1  n1
un1  un

un 2  1 un1  1 un  1
Chia vế của (1) cho (2) có  . , n  N * .
un 2  1 un1  1 un  1

un  1
Đặt vn  n  N * , ta có vn2  vn1.vnn  N * .
un  1

Bằng phương pháp quy nạp ta chứng minh được vn  v2Fn1 .v1Fn2 , với  Fn  là dãy số Phibonxi:
 F1  F2  1
 .
 n2
F  Fn 1  Fn , n  N *

Fn1 Fn2
1  1
Hay vn    .    0 khi n   , dẫn đến lim un  1 .
2  3

Trang26
Bài 3. Cho dãy số  un  được xác định như sau.

u1  1

 .
un 1  un  un  2  un  4  un  6   16, n 
*

n
1
Đặt vn   , hãy tính lim vn .
i 1 ui  5

Hướng dẫn giải

Dễ thấy un  0, n  *
.

Theo bài ra ta có.

u  6un  un2  6un  8   16  u  6un  4   un2  6un  4 .


2
un 1  2
n
2
n

1 1 1
Suy ra un1  1   un  1 un  5    .
un1  1 un  1 un  5
n
1 n
 1 1  1 1 1 1
Do đó vn          .
i 1 ui  5 i 1  ui  1 ui 1  1  u1  1 un 1  1 2 un 1  1
Mặt khác, từ un1  un2  6un  4 ta suy ra un 1  6un .
Kết hợp với u1  1 ta có.
1
un  6n1 , n  *
 lim un    lim  0.
un1  1

1 1  1
Từ đó ta có lim vn  lim    .
 2 un 1  1  2

Bài 4. Cho dãy số thực  un  với n  *


 
thỏa mãn ln 1  un2  nun  1, n  *
.

n 1  nun 
Tìm lim .
n  un

Hướng dẫn giải

Với mỗi n  *
, đặt f n  x   ln 1  x 2   nx  1, x  .

 x  1  n  1  0 .
2
2x
Ta có f  x  
'
 n 
1  x2 1  x2
n

 x  1
f n'  x   0   .
n  1
Do đó f n  x  là hàm tăng thực sự trên .

 f n  0   1  0

Ta có   1   1 .
 f n  n   ln 1  n2   0
    

Trang27
sao cho f n  un   0 và 0  un 
1
Do đó !un  .
n
Ta thấy lim un  0 .
n 

 1

 lim ln 1  un  n  1
2 u2

Do đó:  n .
 lim nun  lim 1  ln 1  un2    1
 n n   

n 1  nun  n ln 1  un  
2
2 u2 
1
Vậy lim  lim  lim  nun ln 1  un  n   1. .
n  n  n 
un un  

 4
a1  3
Bài 5. Cho dãy số  an  thỏa mãn:  n  1, n  .
 n  2  a  n a   n  1 a a
2 2
 n n 1 n n 1

Tìm lim an .

Hướng dẫn giải

 n  2
2
n2
Dễ thấy an  0, n  *
. Từ giả thiết ta có    n  1 .
an1 an

1 1
Với mỗi n  *
, đặt yn   ta có y1  1 và.
an 4

 1 2 1 n2
 n  2              
2 2 2
 n1
y  n  n
y  n 1 n 2 y n y y y .
 n  2
n 1 n n 1 2 n
 4  4

4n2  n  1
2
 n 1   n  2   1 
2 2 2
4
Do đó yn      ...   y1   an  .
 n  1   n 1   3   n  1 n2 16  n2  n  1
2 2

Vậy lim an  4 .

Bài 6. Tính các giới hạn sau:

x3  8 2x 1
a) lim . b) lim .
x 2 x2  4 x2 x2
Hướng dẫn giải

a).lim 2
x3  8
 lim
 x2  2 x  4
 3.
x 2 x  4 x 2  x  2
2x 1
b) lim   .
x2 x2

x  x 2  ...  x n  n
Bài 7. Tính giới hạn lim .
x 1 x 1
Hướng dẫn giải

Trang28
x  x 2  ...  x n  n ( x  1)  ( x 2  1)  ...  ( x n  1)
lim  lim
x 1 x 1 x 1 x 1
( x  1)[1  ( x  1)  ( x  x  1)  ...  ( x n 1  ...  1)]
2
lim .
x 1 x 1

lim 1  ( x  1)  ( x2  x  1)  ...  ( x n1  ...  1)  .


x 1

n(n  1)
 1  2  3   n 
2 .
n
1  ax  1 a
Bài 8. Cho n là số nguyên dương và a  0 .Chứng minh rằng: Lim  ..
x 0 x n
Hướng dẫn giải

Đặt y  n 1  ax, khi đó từ x  0  y  1. .


n
1  ax y 1 y 1 a
Vậy Lim  aLim n  a Lim  ...  . .
x 0 x y 1 y  1 y 1
 y  1  y  y  ...  y  
n 1 n  2
n

Bài 9. Tính các giới hạn sau:.


1
13  53  93  ...  (4n  3)3  cos5 x  x sin x
a/ lim b/ lim   .
1  5  9  ...  (4n  3)  
n  2 x 0 cos3 x

Hướng dẫn giải


Câu a.
n n
13  53  93  ...  (4n  3)3   (4i  3)3   (64i3  144i 2  108i  27) .
i 1 i 1

n n n
= 64 i3  144 i 2  108 i  27n .
i 1 i 1 i 1

n(4n  2)
1  5  9  ...  (4n  3)   2n 2  n .
2

n(n  1) n 2 n(n  1)(2n  1) n 3  n(n  1) 


n 2

Mà ta có các công thức:  i  ; i  ; i   .


i 1 2 i 1 6 i 1  2 

Do đó: P( x)  1  5  9  ...  (4n  3) là một đa thức bậc 4 có hệ số bậc 4 là 64 / 4  16 .


3 3 3 3

Và Q( x)  1  5  9  ...  (4n  3) là một đa thức bậc 4 có hệ số bậc 4 là 4 .


2

13  53  93  ...  (4n  3)3 16


Do đó: lim   4.
1  5  9  ...  (4n  3)
n  2
4

Câu b.

Trang29
cos5 x cos3 x
1
 cos3 x
 x sin x.cos3 x
 cos5 x   cos5 x  cos3 x  cos5 x  cos3 x
= lim 1  
x sin x
lim   x 0 
 
.

x 0 cos3 x
  cos3x 
 
cos5x  cos3x 2sin 4 x sin x  sin 4 x sin x 8 
Vì lim  lim  lim  . .  8 .
x 0 x sin x.cos3x x 0 x sin x.cos3 x x 0
 4x x cos3x 
1
cos 5 x  cos 3 x 1
 cos5 x  x sin x
Vì lim  0 và áp dụng công thức lim 1  u  u  e , nên lim    e8 .
x 0 cos 3 x u 0 x 0
 cos3x 
 x1  2

Bài 10. Cho dãy số  xn  thỏa mãn  x1  2 x2  3x3  ...  (n  1) xn1 . Tìm lim un với
 xn  n(n  1)
2
, n  1, n  .

un  (n  1)3 xn . .

Hướng dẫn giải


1
Ta có x2  .
3

Với n  3 : x1  2 x2  3x3  ...  nxn  n3 xn (1).

x1  2 x2  3x3  ...  (n 1) xn1  (n 1)3 xn1 (2).

Từ (1) và (2) ta có nxn  n3 xn  (n 1)3 xn1 .

(n  1)3 xn 1 n 1 2 n
Suy ra xn  ( ). .xn 1 .
n n
3
n n 1
n 1 2 n  2 2 2 2 n n 1 3
 xn  ( ) .( ) ...( ) . . ... x2 .
n n 1 3 n 1 n 4

4 4(n  1) 2
 xn  suy ra lim u = lim  4.
n 2 (n  1)
n
n2

3x  1. 3 2  x  2
Bài 11. Tính giới hạn hàm số : L  lim .
x 1 x 1
Hướng dẫn giải
Ta có:.

3x  1. 3 2  x  2 3x  1. 3 2  x  3x  1  3x  1  2
lim  lim .
x 1 x 1 x 1 x 1
3
2  x 1 3x  1  2
= lim 3x  1  lim .
x 1 x 1 x 1 x 1

( 3 2  x  1)  3 (2  x) 2  3 2  x  1
= lim 3x  1    lim ( 3x  1  2)( 3x  1  2) .
x 1
( x  1)  3 (2  x) 2  3 2  x  1 x 1 ( x  1)( 3x  1  2)
 

Trang30
(2  x  1) (3x  1  4)
= lim 3x  1  lim .
x 1
( x  1)  3 (2  x) 2  3 2  x  1 x 1 ( x  1)( 3 x  1  2)
 

( 3x  1) 3 1
= lim  lim = .
x 1  3 (2  x) 2  3 2  x  1 x 1 ( 3 x  1  2) 12
 

x 2  3  2011x  2009
Lim
Bài 12. Tính: x 1 x 1 .

Hướng dẫn giải


x 2  3  2  2011( x  1) x2  3  4
lim  lim[  2011]
x 1 x 1 x 1 ( x  1)( x  3  2)
.
x 1 4021
 lim(  2011)  
x 1 x3 2 2

 4
a1  3
Bài 13. Cho dãy số  an  thỏa mãn:  n  1, n  . Tìm lim an .
 n  2  a  n a   n  1 a a
2 2
 n n 1 n n 1

Hướng dẫn giải


 n  2
2
n2
Dễ thấy an  0, n  *
. Từ giả thiết ta có    n  1 .
an1 an

1 1
Với mỗi n  *
, đặt yn   ta có y1  1 và.
an 4
 1 2 1 n2
 n  2  yn1    n  yn     n  1   n  2  yn1  n yn  yn1 
2 2 2
y .
 n  2
2 n
 4  4

4n2  n  1
2
 n 1   n  2   1 
2 2 2
4
Do đó yn      ...   y1   an  .
 n  1   n 1   3   n  1 n2 16  n2  n  1
2 2

Vậy lim an  4 .

1 a
Bài 14. Cho dãy số  xn  thỏa mãn x1  0, xn  (3xn1  3 ), n  2,3,... .
4 xn1

Hướng dẫn giải


1 a
Ta có xn  ( xn1  xn1  xn1  3 )  4 a với mọi n  2 .
4 xn1

Do đó dãy  xn  bị chặn dưới.

xn 3 a 3 1
Với mọi n  3 , ta có   4    1  xn  xn –1 .
xn1 4 4 xn1 4 4

Do đó  xn  là dãy giảm.

Trang31
Từ đó suy ra dãy  xn  có giới hạn và dễ dàng tìm được lim xn  4 a .

 x1  3

Bài 15. Cho dãy số thực  xn  :  1 . Xét dãy số  yn  cho bởi :
 xn 1  3  , n  1, 2,3,...
 xn

(3  5) n
yn  ; n  1, 2,3,... Chứng minh dãy số  yn  có giới hạn hữu hạn và tính giớn hạn đó.
2n.x1.x2 .x3 ...xn

Hướng dẫn giải


1
 Ta có : xn1  3   xn .xn1  3xn  1 ; n  1, 2,3,... .
xn

 Đặt : zn  x1.x2 .x3 ...xn thì ta có zn  2  x1.x2 .x3 ...xn .xn 1.xn  2 .

 zn .xn 1.xn  2 .

 zn .(3xn 1  1) .

 3zn xn 1  zn .

 3zn 1  zn .

 z1  x1  3

 8
Khi đó :  z2  x1.x2  3.  8 . Suy ra  zn  là dãy truy hồi tuyến tính cấp 2.
 3
 zn  2  3zn 1  zn ; n  1, 2,3,...

3 5
Xét phương trình đặc trưng : t 2  3t  1  0  t  .
2
n n
 3 5   3 5 
Dãy có số hạng tổng quát dạng zn         .
 2   2 

 3  5   3 5 
        3  53 5
 
 2   2   10
trong đó :   .
 7  3 5   73 5    5  3 5
 2     2    8  10
   
 Lúc này, ta có.
n n
 3 5   3 5 
   
(3  5) n
 2   2  1
yn  n   n n
 n
.
2 .x1.x2 .x3 ...xn zn  3 5   3 5   3 5 
      
 2   2   3 5 

Trang32
1 1 1 3 5 5
Suy ra : lim yn     .
 3 5 
n
 53 5 2
 .lim    10
 3 5 

3 5 5
 Vậy : yn  khi n   .
2
un
Bài 16. Cho dãy số  un  xác định bởi: u0  1 , un 1  n  . Tìm lim n3un  ? .
n un  u n  1
2 2 n 

Hướng dẫn giải


n
un un 1
Từ giả thiết un 1  n  ta có un1   2 n  *
nên  vn  xác định bởi vn   uk có
n un  un2  1
2 2
n un n k 0

giới hạn hữu hạn, giả sử lim vn  c ( c hữu hạn).


n 

un 1 1
Cũng từ un 1  n  ta có  n2  un  n  .
n un  un2  1
2
un1 un

1 1
   n2  un n  .
un1 un

1 1
Do đó   02  u0 .
u1 u0

1 1
  12  u1 .
u2 u1

….
1 1
  (n  1)2  un1 .
un un1

1 1 (n  1)n(2n  1) n 1
Cộng theo vế ta được :     uk .
un u0 6 k 0

1 (n  1)n(2n  1) vn1  1
 3
  .
n un 6n3 n3

1  vn
Mà lim  0 ( do lim vn  c ) nên.
n  n 3 n 

1 (n  1)n(2n  1) 1
 lim 3
 lim 3
 hay lim n3un  3 .
n  n un n  6n 3 n 

4
Bài 17. Cho dãy số  xn  xác định bởi : x1  1, xn1  1  , n  1 . Chứng minh dãy  xn  có giới hạn hữu
1  xn
hạn và tìm giới hạn đó.

Hướng dẫn giải

Trang33
4 4 4
Ta có x2  1   3; x3  1   2  x1 ; x4  1   x2 .
2 4 3
4
Hàm số f ( x)  1  liên tục và nghịch biến trên [0,+), 1  f ( x)  5 .
1 x

4
Ta có xn1  1   f ( xn ), n  ( xn ) bị chặn.
1  xn

x1  x3  f ( x1 )  f ( x3 )  x2  x4  f ( x2 )  f ( x4 )  x3  x5  ... .

suy ra dãy ( x2 n 1 ) tăng và dãy ( x2 n ) giảm suy ra ( x2 n 1 ), ( x2 n ) là các dãy hội tụ.

Giả sử lim x2 n  a;lim x2 n 1  b (a, b  1) .

Từ x2 n 1  f ( x2 n )  lim x2 n 1  lim f ( x2 n )  b  f (a) .

Từ x2 n  2  f ( x2 n 1 )  lim x2 n  2  lim f ( x2 n 1 )  a  f (b) .

 4
b  1  1  a
Giải hệ phương trình   a  b  4  2 . Vậy lim xn  2 .
a  1  4
 1 b
1 xn
Bài 18. Cho x1  2014, x2  2013 và xn  2  (1  ) xn 1  , n  2, 3,... Tìm lim xn .
n n n

Hướng dẫn giải

xn 1  xn (1) n (1) n n
(1) k
Ta có xn  2  xn 1    xn  2  xn 1  ( x2  x1 )   và xn  2  x1   .
n n! n! k 1 k!

(1) k 
(1) k 1
Dãy này rõ ràng hội tụ và có giới hạn là x1    x1  1    x1  1  .
k 1 k! k 0 k! e

1
Từ đó suy ra lim xn  2015  .
n  e

3.3. TÍNH GIỚI HẠN BẰNG ĐỊNH LÍ KẸP


1
Bài 1. Tìm lim n
.
n n!
Hướng dẫn giải
n n
Trước hết ta chứng minh bất đẳng thức : n! > ( ) (*) ( n  N*).
3
1
Bằng phương pháp qui nạp. Thật vậy : với n =1, ta có 1 > (đúng).
3
k k.
Giả sử (*) đúng với n = k tức là : k! > ( ) Ta đi chứng minh (*) đúng với.
3

Trang34
n = k+1.
k k k  1 k+1 3 k  1 k+1
Ta có (k+1)! = k!(k+1) >( ) (k+1) = ( ) . >( ) .
3 3 1 k 3
(1  )
k
Bất đẳng thức cuối này đúng vì :.
1 k k k ( k  1) 1 k (k  1)(k  2)....(k  k  1) 1
(1+ ) =1+ + . 2 +.+ . k =.
k k 2! k k! k
1 1 1 1 2 k 1 1 1 1 1
= 1+1+ (1  ) +.+ (1  )(1  )...(1  ) < 1+1+ +… + <1+1+ +.+ n1 <.
2! k k! k k k 2! n! 2 2
1 1 1
<1+1+ +.+ n1 +.< 1+ = 3.
2 2 1
1
2
n
n n
Vậy (*) đúng với n  k  1 . Do đó n !    , từ đây ta suy ra n
n!  .
3 3

1 3
=> 0 < n
< .
n! n
3
Vì lim = 0.
n n
1
Do đó theo định lý về giới hạn kẹp giữa ta suy ra: lim = 0.
n n n!
1
Vậy lim(2014  n
) =2014.
n!

 x1  1; x2  2

Cho dãy số  xn   xn 1 
5
thoả mãn  .
 xn  2   ; n 
2 *

 
2
 4 x n

Tính I  lim xn .

Từ giả thiết suy ra mội số hạng của dãy đều dương.


 y1  0; y2  1
Đặt yn  log 2 xn , ta có dãy  .
2 yn  2  5 yn 1  2 yn ; n 
*

 z1  2, z 2  1
 z1  2; z2  1 
Lại đặt yn  zn  2 , ta có dãy  2 z n  2  5 z n 1  z n .
2 zn  2  5 zn 1  zn ; n 
*

1
Tìm được số hạng tổng quát của dãy là zn  4. .
2n
Từ đó ta có lim yn  2  lim xn  4 .

Trang35
an2  5an  10
Bài 2. Cho dãy (an )n 1 : a1  1; an1  , n  1 .
5  an
a) Chứng minh dãy (an ) hội tụ và tính lim an .

a1  a2  ...  an 5  5
b) Chứng minh  , n  1. .
n 2
Hướng dẫn giải
3
a) Bằng phương pháp chứng minh qui nạp ta có: 1  an  , n .
2

5 5 x 2  5 x  10 10
Đặt A  và xét hàm f ( x)    x, ( x  5) .
2 5 x 5 x
10  3 1 
Suy ra f ( x)   1  0, x  1;  . , như vậy f ( x ) nghịch biến trên đoạn  ;1 . .
5  x   2 2 
2

a  a3  a5  ...  a2 k 1  ...  A  lim a2 k 1  b  A


Dẫn đến  1  ..
 a2  a4  a6  ...  a2 k  ...  A   lim a2 k  c  A
Kết hợp công thức xác định dãy ta được.

 c 2  5c  10
 b 
5c 5 5
 bc .
c  b  5b  10
2
2
 5b

5 5
Vậy lim an  ..
2
 5 5 
b) Nhận xét: t  1;  . thì t  f (t )  5  5. .
 2 

Dẫn đến a2 k 1  a2 k  5  5 , k  1 .

5 5
 a1  a2  ...  a2 k 1  a2 k  2k . (1).
2
Như vậy bất đẳng thức đúng với n  2k .

5 5
Trường hợp n  2k  1, chú ý a2 k 1  , kết hợp với (1) thu được:.
2

5 5
a1  a2  ...  a2 k 1  a2 k  a2 k 1  (2k  1) .
2
Vậy bất đẳng thức được chứng minh.

1 u e un
Cho dãy số thực  un  : u1  , un 1  n un , n  *
Bài 3. .Chứng minh dãy trên có giới hạn hữu hạn,
2 1 e
tìm giới hạn đó.

Trang36
Hướng dẫn giải

Chứng minh 1  un  0, n  2 1 .

1
e
Với n  2, u2  2   1;0  đúng.
1 e

Giả sử 1 đúng với n  k  2 , ta chứng minh 1 đúng với n  k  1 .

u n e un
Ta có un  0  eun  1  1  eun  0  0.
1  e un

1 u n e un
u n  1  e u n  ;  1  un eun  eun  1  eun  un  1  1 (luôn đúng).
e 1  e un
Vậy (1) được chứng minh.

xe x e x 1  x  e x 
Xét hàm f  x   trên  ;0  . Ta có f '  x   .
1  ex 1  e x 
2

Hàm g  x   1  x  e x có g '  x   1  e x  0 với mọi x   ;0  nên hàm này đồng biến trên  ;0  .

e x 1  x  e x 
Suy ra g  x   g  0   0 , suy ra f '  x    0.
1  e x 
2

hay hàm f  x  nghịch biến trên  ;0  .

e
e 
2 1 e 
 
1 e
e
e 2 1  e
Ta có u2  2  , u3  , u4  u2 .
1 e 2 1 e  
1 e
e
21 e 

Suy ra f  u4   f  u2   u5  u3  0  u1 .

Quy nạp ta được dãy  u2 n 1  giảm và dãy  u2n  tăng.

Hơn nữa 1  un  0, n  2 nên mỗi dãy trên tồn tại giới hạn hữu hạn.

Giả sử lim u2 n  a, lim u2 n 1  b  a, b   1;0   , lấy giới hạn hai vế ta được.

 beb
 a  aea
1  eb
 a
 1  e   e e .
a 2 a 1 ea

b  ae
 1  e a

  1  t
Đặt e  t  t   ;1  , ta được phương trình 1  t   t.t  2 1  t  ln 1  t   1  t  ln t  t ln t  0 .
a 2 1t

  e 

Trang37
Hàm h  t   2 1  t  ln 1  t   1  t  ln t  t ln t nghịch biến nên phương trình có nhiều nhất 1 nghiệm,
1
nhận thấy t  là nghiệm nên nó là nghiệm duy nhất.
2
1 1
Suy ra a  ln , thay vào được b  ln .
2 2
1
Vậy lim un  ln .
2
2n  3
Bài 4. Cho dãy số  an  , n  1 thỏa mãn a1  1, an  an 1 , n  2 và dãy  bn  , n  1 thỏa mãn
2n
n
bn   ai , n  1 . Chứng minh dãy  bn  có giới hạn và tìm giới hạn đó.
i 1

Hướng dẫn giải

Ta có 2nan   2n  3 an1  an1  2  n  1 an1  nan  , n  1 .


n
Do đó bn   2 iai   i  1 ai 1   2 1   n  1 an 1  .
i 1

1
Ta chứng minh bằng quy nạp rằng nan  , n  1.
n
Thật vậy:.
- Với n = 1, ta có a1  1 nên khẳng định đúng.

2  n  1  3  2n  1   1 
- Giả sử khẳng định đúng với n  n  1 . Ta có an 1  an     , ta cần chứng
2  n  1  2n  2   n n 
 2n  1   1  1
minh       2n  1 n  1  2n n .
 2n  2   n n   n  1 n  1

  4n2  4n  1  n  1  4n3  1  3n .

Bất đẳng thức cuối đúng nên khẳng định trên đúng với n  1.
Theo nguyên lí qui nạp thì khẳng định được chứng minh.

 1 
Ta có 2 1    2 1   n  1 an1   bn  2 .
 n 1 

Theo nguyên lí kẹp thì dãy  bn  có giới hạn và lim bn  2 .

 1
u1  2

Bài 5. Cho dãy số  bn  được xác định bởi:  .
 
un 1  1  un  un 2  1 
 2  4n 

Chứng minh dãy số hội tụ và tìm lim un .


x 

Trang38
Hướng dẫn giải
1 
Ta chứng minh un  n
cot n 1 ; n  (*) .
2 2
1  1
Thật vậy: n  1 : u1  1
cot 11  .
2 2 2
 (*) đúng với n  1 .
1 
Giả sử (*) đúng tới n  k , k  *
, nghĩa là có : u  cot .
2k  1
k
2k

1 1 
Ta chứng minh (*) cũng đúng với n= k+1. Thật vậy uk 1   uk  uk 2  k  .
2 4 

1 1  1  1  1   2 

  k cot k 1  k cot 2 k 1  k   k 1  cot k 1  cot k 1  1  .
22 2 4 2 4  2  2 2 

 
1   1  
 k 1  cot k 1 
  ( vì khi k   thì k 1  0; sin  0 ).
2  2 sin k 1  2
 2 
 
cos k 1  1 2 cos 2 k  2
1 2 1 2 1 
 k 1 .  k 1  k 1 cot k  2 .
2  2 2sin  cos  2 2
sin k 1 k 2 k 2
2 2 2
 (*) cũng đúng với n  k  1 .
1 
Vậy un  n
cot n 1 ; n  .
2 2

     
 cos 2n 1 1    2 n 1
2  2
lim un  lim  . n   lim  cos n 1  .
x  x 

 2  x 
 2    
n 1
 2   2n 1 
2
Vậy dãy hội tụ và có lim un  .
x  
Bài 6. Cho phương trình: x n  x 2  x  1  0 với n N, n  2 .

1)Chứng minh rằng với mỗi số nguyên n  2 , thì phương trình có một nghiệm dương duy nhất xn .
2)Xét dãy số sau đây: U n  n  xn  1 , n  2,3, 4,... Tìm limU n ? .
Hướng dẫn giải

Xét phương trình: f  x   x n  x 2  x  1  0 , với n nguyên, n  2 (1).

+) Ta có: f ’  x   nx n1 – 2 x –1 . Do n  2 , nên khi x  1 thì f ’  x   0 . Vậy f  x  là hàm số đồng biến


trên 1;  .

Trang39
Lại có: f 1  2  0 ; f  2   2n – 7  0 ( vì n nguyên và n  2  n  3).

Ta có: f 1 f  2   0 và f  x  liên tục, đồng biến nên phương trình f  x   0 có nghiệm duy nhất trên
1; .
+) Mặt khác với 0  x  1 thì x n  x 2 ( do n  2 ) suy ra f  x   0 với mọi 0  x  1 .

Như vậy ta đã chứng minh được (1) có nghiệm dương duy nhất với mọi n nguyên, n  2 .
Gọi xn là nghiệm dương duy nhất của phương trình x n – x 2 – x –1  0 .

Bây giờ xét dãy U n  với U n  n  x n  1 , n  3, 4,5, .

Ta có: xnn  xn2  xn  1  0 hay xn  n xn2  xn  1 .

Áp dụng bất đẳng thức AM – GM, ta có:.

xn2  xn  1

1  ....
 1
xn  n xn2  xn  1  n xn2  xn  1.1
n so 1
.1
....
1 < (2).
n 1 sô 1 n

(Chú ý rằng ở đây 1  xn nên xn2  xn  1  1 , vì thế trong bất đẳng thức không có dấu bằng).

6
+) Mặt khác do xn  2 , nên xn2  xn  6 , nên từ (2) có: 1  xn  1  (3).
n
6
Bất đẳng thức (3) đúng với mọi n  3 và lim  0 nên từ (3) ta có: lim xn  1 .
n

lnxn2  xn  1

+) Ta có: xnn  xn2  xn  1  n ln xn  ln xn2  xn  1  n   ln xn
.

Từ đó: nxn  1 
xn  1 lnx 2  x  1 (5).
n n
ln xn

Đặt y n  x n  1  lim y n  0 .

Ta có: suy ra từ (5) lim U n  lim n  xn  1  ln 3 .


Vậy: lim U n  ln 3 .

ln xn ln  yn  1 ln  t  1
Bài 7. Cho số thực a, xét dãy số  xn n 1 được lim  lim  lim  1 xác định bởi
xn  1 yn t 0 t
xn3  6 xn  6
x1  a, xn1  , n  1, 2,.... .Tìm tất cả các giá trị của a để dãy số có giới hạn hữu hạn,
3xn2  9 xn  7
tìm giới hạn đó?.

Hướng dẫn giải


Với a  1 thì xn  1, n  1 nên lim xn  1 .
n 

 xn1  1  xn1  2
3 3

Với a1 thì xn  1  , xn  2  , n  2 .


3xn21  9 xn1  7 3xn21  9 xn1  7

Trang40
3 3n1
x  2  xn 1  2  a2
Do đó n    ...   , n  1 .
xn  1  xn 1  1   a 1 
3n1 3n1
2  a  1   a  2
Từ đó, tính được xn  3n1 3n1
, n  1 ,.
 a  2   a  1

3
Kết luận + a    a  1  a  2  lim xn  2 .
2 n 

3
+ a    a  1  a  2  lim xn  1 .
2 n 

3 3 3
+ a    xn   , n  1  lim xn   . .
2 2 n  2

 2012
u1 
Bài 8. Cho dãy số (un ) xác định như sau:  2013 . Tìm lim un .
n 
u  2u  1  0 , n  1, 2,3,...
2
 n n 1

Hướng dẫn giải


un2 1
Ta có : un 2  2un 1  1  0  un 1   .
2 2

x2 1 x2 1 1
Xét hàm số : f ( x)     .
2 2 2 2
f '( x)  x .

. x 1 0 1
2
f  x
Ta có :.  0 
f  x 3 0
8

1
2

1 1 1 3
 u1  1   u2  0   u3    0 .
2 2 2 8
Vậy : n  2 thì 1  un  0 .

un2  1
un 2  2un 1  1  0  un 1  .
2

x2 1 1
Gọi a là nghiệm của :  x ( x  ( ;0))  a  1  2 .
2 2

Ta có : un1  a  f (un )  f (a) .

Trang41
Theo định lí La-grăng : f (un )  f (a)  f '(a) . un  a .

1 1
Do f '(a )   f (un )  f ( a )  un  a .
2 2
2 n
1 1 1
 un1  a  un  a    un1  a  ...    u1  a .
2 2 2
n
1
Mà lim    0  lim (un1  a)  0  lim un1  a  1  2 .
 
n  2 n  n 

Vậy : lim un  1  2 .
n 

 1
u0  2

Bài 9. Cho dãy số un  xác địnhnhư sau:  .Chứng minh rằng dãy số un  có giới
un 1  un  5 , n 
2

 2  un  2 
hạn và tìm giới hạn đó.

Hướng dẫn giải


* Vì 0  u0  1 nên 0  un  1, n  .

9
* Áp dụng BĐT Cauchy ta có un  2   6 . Dấu bằng xảy ra  un  1 .
un  2
9
 un  2   6 , n  .
un  2
un 2  5 1 9 
* un 1    un  2    2  1, n  .
2  un  2  2  un  2 

1 9
* un 1  un   un  1  .
2 2  un  2 

1 9
Xét hàm số f  x    x  1  .
2 2  x  2

1 9
f ' x     0, x  1  f  x  nghịch biến trên 1;   .
2 2  x  2 2

* Vì un  1  f  un   f 1  0  un1  un , n  *
.
 un  giảm và bị chặn dưới  un  có giới hạn hữu hạn.

un 2  5
* Giả sử lim un  a 1  a    . Từ un 1  chuyển qua giới hạn ta có.
2  un  2 

a2  5 a  1
a  .
2  a  2  a  5(loai)
* Vậy lim un  1 .

Trang42
un1
Bài 10. Cho dãy số (un ) được xác định bởi: u1  4 và un1  un2  2 , với n  *
.Tìm lim .
n  u .u ...u
1 2 n

Hướng dẫn giải


Với mọi n  1, 2,... ; ta có.

un21  4   un2  2  4  un4  4un2  un2 un2  4   un2 .un21 (un21  4) .


2

 ...  un2un21...u22u12 (u12  4)  12  un .un1...u1  (1).


2

2
 un 1  4
Từ (1) ta có:    12  ; n  1, 2,... (2).
 u1.u2 ...un 
2
 u1.u2 ...un 

Mặt khác, vì u1  4  2 nên từ un1  un2  2 và chứng minh bằng quy nạp ta thu được un  2 với mọi
n  1, 2,... .

4 4
Do đó u1.u2 ...un  2n ; n  *
. Khi đó, 0   ; n  1, 2,... .
 u1.u2 ...un 
2
22 n

4
nên theo nguyên lý kẹp giữa ta có: lim  0.
 u1.u2 ...un 
n  2

2
 un 1 
Vậy, từ (2) suy ra: lim 
n  u .u ...u
  12 .
 1 2 n

Mặt khác, hàm số f ( x)  x liên tục trên nửa khoảng [0;  ) nên.
2 2
un 1  un 1   un 1 
lim  lim    lim    12 .
n  u u ...u n  n  u u ...u
1 2 n  u1u2 ...un   1 2 n
un1
Kết luận: lim  12 .
n  u1.u2 ...un

Bài 11. a) Chứng minh rằng có đúng một dãy số thực ( xn ) n 0 thỏa mãn.

xn  xn 1
x0  1, 0  xn  1n  1 và (1  xn ) 2  (1  xn 1 ) 2  n  1. .
2
b) Với dãy ( xn ) xác định như trên, xét dãy ( yn ) n0 xác định bởi yn  x0  x1  ...  xnn  0. Chứng
minh rằng dãy ( yn ) n0 có giới hạn hữu hạn khi n   . Hãy tìm giới hạn đó.

Hướng dẫn giải


a) Bằng quy nạp ta sẽ chỉ ra rằng xn xác định duy nhất với mỗi n  0. Để làm được điều này ta cần dùng
kết quả (chứng minh của nó là đơn giản) sau: Với mỗi số thực m  [0;1] , phương trình
tm
(1  t ) 2  (1  m) 2  có đúng một nghiệm trên [0;1] .
2
1 1 1 1 1
b) Để ý rằng yn  x0  ( x0  x1 )  ( x1  x2 )   ( xn 1  xn )  xn n  1. .
2 2 2 2 2
Trang43
3
Ta có giới hạn cần tìm bằng ..
2

Bài 12. Giả sử  Fn  n  1, 2,... là dãy Fibonacci ( F1  F2  1; Fn 1  Fn  Fn 1 với ). Chứng minh rằng nếu
Fn1 1
a với mọi n  1, 2,3,... thì dãy số  xn  , trong đó x1  a, xn1   n  1, 2,3... , là xác
Fn 1  xn
định và nó có giới hạn hữu hạn khi n tăng lên vô hạn. Tìm giới hạn đó.

Hướng dẫn giải


Giả sử x1 , x2 ,..., xm đã được xác định. Khi đó xm 1 được xác định khi xm  1 .

1
* Nếu xm  1 thì do xm  nên xm 1  2 .
1  xm1

F2 F
Từ giả thiết F1  F2  1; Fn 1  Fn  Fn 1 ta viết xm   , xm1   3 .
F1 F2

Fi  2
Giả sử xmi   , với i nào đó, 0  i  m  2 .
Fi 1

1 1 F F
Vì xmi  nên xmi 1   1   i 1  1   i 3 .
1  xmi 1 xmi Fi  2 Fi  2

Fm1 F
Khi đó x1   . Mâu thuẫn với giả thiết x1   m1 . Như vậy ( xn ) là dãy số xác định.
Fm Fm

1 5 1  5 1
Phương trình x   x 2  x  1  0 có hai nghiệm u  ,v  . Có hai trường hợp xảy ra:.
1 x 2 2

 5 1
Trường hợp 1: x1  v . Khi đó xn  x1 , n  1 . Do đó lim xn  .
n  2
1 1 v
Trường hợp 2: x1  v . Chú ý  v  xn   xn  v . Do đó xn  v, n  1 .
1  xn v

xn  u
Đặt zn  , ta có.
xn  v

1
u
xn 1  u 1  xn (1  u )  uxn u 2  uxn u xn  u u
zn 1     2  .  . zn .
xn 1  v 1
v (1  v )  vxn v  vxn v xn  v v
1  xn
n
u u
Từ đó có zn    .z1 nên zn  0 khi n   (vì  1 ).
v v
xn  u u  vzn
Từ zn  suy ra xn  dần tới u khi n   (do zn  0 ).
xn  v 1  zn

5 1
Tức là trong trường hợp này lim xn  .
n  2

Trang44
 yn 
Bài 13. Cho dãy số  yn  thỏa mãn y1  0, yn31  y1  y2  ...  yn , n  1 . Chứng minh rằng dãy số  
n
có giới hạn bằng 0 khi n   .

Hướng dẫn giải

Từ giả thiết ta có yn31  yn  yn3 , n  2 , do đó dãy số  yn n 2 là dãy tăng, vì.

vậy yn31  yn  yn3  yn ( yn2  1)  yn1 ( yn2  1) .

 yn21  yn2  1 , n  2  yn21  yn2  1  ...  y22  n  1 .

y2  n 1 y22  n  1
2
 y 
  n 1   2 . Mà lim  0 nên theo định lý kẹp ta có.
 n 1  (n  1)2 (n  1) 2
2
 y  y y
lim  n1   0  lim n 1  0  lim n  0 .
 n 1  n 1 n

Bài 14. Cho  un  là một dãy số dương. Đặt Sn  u13  u23  ...  un3 với n  1, 2,... Giả sử

un1    Sn  1 un  un 1 
1
với n  2, 3,.... Tìm lim un .
Sn1

Hướng dẫn giải

Ta có Sn1  Sn  un31  0, n  1, 2,...   Sn  là dãy số tăng.

Nếu dãy số  S n  bị chặn trên thì  S n  là một dãy hội tụ và lim un3  lim  Sn 1  Sn   0  lim un  0 .

Xét trường hợp dãy số  S n  không bị chặn trên thì lim Sn   .

Từ giả thiết ta có Sn 1un 1  un  Snun  un1 , n  2,3,... .

Từ đây ta thu được Snun  un 1  S2u2  u1 , n  2,3,... .

un1 S2u2  u1 S u u
Do đó un    0  un  2 2 1 , n  2,3,... .
Sn Sn Sn

Theo nguyên lí kẹp ta có lim un  0 .

Vậy trong mọi trường hợp ta đều có lim un  0 .

u1  1

Bài 15. Cho dãy số (un ) xác địnhbởi công thức truy hồi:  1 . Chứng minh rằng
u n 1  u n   2, n  *

 un
dãy (un ) có giới hạn hữu hạn và tính giới hạn đó.

Hướng dẫn giải


1 1 1
Đặt f ( x)  x   2; g ( x)  f ( f ( x))  x    2 2 . Khi đó.
x 1
x x  2
x

Trang45
 2 2
2  x    x  1
 2  1 1
g '( x)  2
 0  g ( x)  g ( )  0  f ( f ( x))  x, x  ( ;1) (*). .
 1  2 2
x4  x   2 
 x 
1
Mặt khác f '( x)  0, x  ( ;1) nên.
2
1 1 1 1 1
f ( x)  f ( )  f ( f ( x))  f ( )  , x  ( ;1) (**). .
2 2 2 2 2
1 1
Từ (*) và (**) suy ra:  f ( f ( x))  x, x  ( ;1). .
2 2
1 1
Vậy: 1  u1  u3   1  u1  u3  u5  ,... Do đó (u2 n 1 ) là đơn điệu giảm và bị chặn dưới nên tồn
2 2
1
tại lim u2 n 1  ..
n  2

 1 
Vì f ( x ) liên tục trên  ;1 nên.
 2 

n 

u2 n  f (u2 n 1 )  lim u2 n  f lim u2 n 1 
n 
 1
2
..

Vậy dãy (un ) được phân tích thành hai dãy con hội tụ tới cùng một giới hạn. Do đó dãy (un ) có giới hạn
1
bằng ..
2
Bài 16. Tìm tất cả các hàm số f :  thỏa mãn đồng thời các điều kiện sau đây:.

1. f  x  y   f ( x)  f ( y ) với mọi x, y  .

2. f ( x)  e x  1 với mỗi x .

Hướng dẫn giải


f  x  0   f ( x)  f (0)  f (0)  0 và bởi vì f (0)  e0  1  0 nên f (0)  0 .

f ( x  ( x))  f ( x)  f ( x)  f ( x)  f ( x)  0 (1) .

 x  x  x 
f ( x)  f    f    2  e 2  1 .
2 2  

 x  x  x  x 
f ( x)  2  e 2  1  f ( x)  f    f    4  e 4  1 .
  2 2  

 2xn 
Dùng quy nạp theo n  1, 2,  ta CM được f ( x)  2  e  1 .
 

Trang46
 2x0n 
Cố định x0  ta có f ( x0 )  2  e  1 .
n
 
 

 2x0n 
 n x0
  e 1 
Xét dãy an  2n  e 2  1 ta có : lim an  lim  x0 x0   x0 .
 
   2
n


Vậy f ( x0 )  x0 , x0  (2) .

Vậy f ( x)  f ( x)  x  ( x)  0 (3) .

Kết hợp ( 1) và (3) ta được f ( x)  f ( x)  0 .

Từ (2)  f ( x)   x  f ( x)  x (4) . Kết hợp ( 2) và (4) ta được f ( x)  x, x  .

Thử lại f ( x)  x ta thấy đúng.

 x1  1,

Bài 17. Cho dãy số  xn  được xác định như sau  3 x .Chứng minh rằng  xn  có giới
 xn1  xn  2 n  1
n

 n
hạn hữu hạn khi n dần đến vô cùng.

Hướng dẫn giải


Dễ thấy xn  0 , với mọi n nguyên dương, nên dãy số đã cho là dãy tăng thực sự.

Vậy để chứng minh dãy số có giới hạn hữu hạn ta chỉ cần chứng minh nó bị chặn trên.

Ta chứng minh xn  8, n  *
.

Thật vậy, với n  1  x1  1  8 nên điều cần chứng minh đúng.

Giả sử ta có: xn  8 , với n nguyên dương. Ta cần chứng minh xn 1  8 .

n n
3xk 1
Theo công thức xác định dãy số có: xn1  x1   2
 1  2 2
 1  2.2  8 .
k 1 k k 1 k

Do đó xn  8 với mọi n nguyên dương từ đó suy ra điều phải chứng minh.

 1 3
a1  4 ; a2  10
Bài 18. Cho dãy số thực  an  xác định bởi  2
.Chứng minh rằng dãy  an 
a  1  an  an 1 , n  , n  2
 n 2 6 3
có giới hạn hữu hạn. Hãy tìm giới hạn đó.

Hướng dẫn giải

Có a1 , a2   0;1 , giả sử a1 , a2 ,..., ak   0;1 , k  , k  2 . Từ công thức truy hồi ta có:.

1 1 ak ak21 1 1 1
 0  0  ak 1        1, vì 0  ak 1 , ak  1  ak 1   0;1 .
2 2 6 3 2 6 3

Vậy bằng phương pháp quy nạp ta chứng minh được an   0;1 , n  *
.

Trang47
 1  3
 x  x   y  y 
 
1 2 1 2
4 10
Xét hai dãy số mới  xn  :  2
và  yn  :  2
với n  ; n  2 .
 x   n  n 1
1 x x  y   n  n 1
1 y y


n 1
2 6 3  n 1 2 6 3
1
Có 0  x1  x2   x3  1 , giả sử ta có 0  x1  x2  ...  xk  1, k  , k  3 , khi đó.
2

1 xk 1 xk22 1 xk xk21
xk        xk 1 .
2 6 3 2 6 3

Vậy bằng phương pháp quy nạp ta chứng minh được  xn  là dãy số tăng và bị chặn trên bởi 1, nên nó có
giới hạn hữu hạn lim xn   .

 3
1  2 
Chuyển công thức truy hồi qua giới hạn tìm được      2.
2 6 3 
  1

Do  xn    0;1 nên suy ra   1 .

Chứng minh tương tự đối với dãy số  yn  , ta cũng có lim yn  1 .

Cuối cùng ta chứng minh xn  an  yn , n  *


(1) bằng phương pháp quy nạp:.

Ta có x1  a1  y1 và a2  x2  y2 , với n = 1, 2 bất đẳng thức (1) đúng. Giả sử (1) đúng tới k  , k  2 ,
tức là xi  ai  yi , i  1, 2,..., k . Khi đó.

1 xk xk21 1 a a2 1 y y2
xk 1     ak 1   k  k 1   k  k 1  yk 1 .
2 6 3 2 6 3 2 6 3
Từ xn  an  yn , n  , n  1 và áp dụng định lý kẹp ta suy ra được lim an  1 .

Bài 19. Cho hai dãy số  an  ;  bn  xác định bởi a1  3, b1  2 , an1  an2  2bn2 và bn 1  2anbn với n = 1, 2,

3,….Tìm lim 2n bn và lim 2n a1a2 ...an .


n n

Hướng dẫn giải


Với mọi n = 1,2,3,… ta có.

 
2
an1  bn1 2  an2  2bn2  2 2anbn  an  bn 2 .

Do đó:.

         
2 22 2n1 2n1 2n
an  bn 2  an 1  bn 1 2  an  2  bn  2 2  ...  a1  b1 2  3 2 2  2 1 .

 
2n
Tương tự ta có: an  bn 2  2 1 .

1
     1 
    
2n 2n 2n 2n
Từ đó: an   2 1 2 1  ; bn   2 1 2 1 .
2  2 2 

Trang48
   
2n 2n

2n
2 1 n
2 1
Chú ý:  2n bn  2n an  2  1 và lim 2  2  1 , nên theo nguyên lí kẹp ta có:
4 2 n 4 2
lim 2n bn  lim 2n an  2  1 .
n n

bn1 b b b b
Mặt khác: bn 1  2anbn hay an  (n  1) . Suy ra: a1a2 ...an  2 . 3 ... n1  nn1 . Do đó
2bn 2b1 2b2 2bn 2

  1
2
lim 2n a1a2 ...an = lim 2n bn 1  2  1  3  2 2 (vì lim 2n  1 ).
n n  n 2n

 1 3
 a1  ; a2 

Cho dãy số thực  an  xác định bởi  .Chứng minh rằng dãy  an 
4 10
Bài 20. 2
a  1  an  an 1 , n  , n  2


n
2 6 3
có giới hạn hữu hạn. Hãy tìm giới hạn đó.

Hướng dẫn giải

+ Ta Có a1 , a2   0;1 , giả sử a1 , a2 ,..., ak   0;1 , k  , k  2 . Từ công thức truy hồi ta có:.

1 1 a a2 1 1 1
 0  0  ak 1   k  k 1     1, vì 0  ak 1 , ak  1  ak 1   0;1 .
2 2 6 3 2 6 3

Vậy bằng phương pháp quy nạp ta chứng minh được an   0;1 , n  *
.

 1
 x1  x2  4
+ Xét hai dãy số mới  xn  :  2
.
x    1 xn xn 1
, n  , n  2
 n 1 2 6 3

 3
 y1  y2  10
và  yn  :  2
.
 y  1  yn  yn 1 , n  , n  2
 n 1 2 6 3

1
- Có 0  x1  x2   x3  1 , giả sử ta có 0  x1  x2  ...  xk  1, k  , k  3 , khi đó.
2

1 xk 1 xk22 1 xk xk21
xk        xk 1 .
2 6 3 2 6 3

Vậy bằng phương pháp quy nạp ta chứng minh được  xn  là dãy số tăng và bị chặn trên bởi 1, nên nó có
giới hạn hữu hạn lim xn   . Chuyển công thức truy hồi qua giới hạn tìm được
 3
1  2  
    2 . Do  xn    0;1 nên suy ra   1 .
2 6 3 
  1

- Chứng minh tương tự đối với dãy số  yn  , ta cũng có lim yn  1 .

Trang49
- Cuối cùng ta chứng minh xn  an  yn , n  *
(1) bằng phương pháp quy nạp:.

Ta có x1  a1  y1 và a2  x2  y2 , với n = 1, 2 bất đẳng thức (1) đúng. Giả sử (1) đúng tới k  , k  2 ,
tức là xi  ai  yi , i  1, 2,..., k . Khi đó.

1 xk xk21 1 ak ak21 1 yk yk21


xk 1     ak 1        yk 1 .
2 6 3 2 6 3 2 6 3
+ Từ xn  an  yn , n  , n  1 và áp dụng định lý kẹp ta suy ra được lim an  1 .

1
Bài 21. Tìm giới hạn: lim(2014  n
).
n!
Hướng dẫn giải
n
n
Trước hết ta chứng minh bất đẳng thức: n !    (*) n  N * ).
3
1
Bằng phương pháp qui nạp. Thật vậy: với n  1 , ta có 1  (đúng).
3
k
k
Giả sử (*) đúng với n  k tức là: k !    . Ta đi chứng minh (*) đúng với.
3
n  k 1 .

k  1 k 1 k  1 k 1
k
k 3
Ta có  k  1!  k ! k  1     k  1  ( ) . ( ) .
3 3 1 k
(1  ) 3
k
Bất đẳng thức cuối này đúng vì:.

k k (k  1) 1 k (k  1)(k  2)....(k  k  1) 1
k
 1
1    1   . 2  ...  . 2
 k k 2! k k! k
1 1 1 1 2 k 1
 1  1  (1  )  ...  (1  )(1  )...(1  )
2! k k! k k k
1 1 1 1 1 1
 1  1   ...   1  1   ...  n 1  1  1   ...  n 1  ..
2! n! 2 2 2 2
1 .
 1 3
1
1
2
n
n n
Vậy (*) đúng với n  k  1 . Do đó n !    , từ đây ta suy ra n
n! > .
3 3

1 3 3
=> 0  n
 . Vì lim 0.
n! n n n

1
Do đó theo định lý về giới hạn kẹp giữa ta suy ra: lim = 0.
n n n!

Trang50
1
Vậy lim(2014  n
) =2014.
n!

3.4. CÁC DẠNG KHÁC

 x1  2016

Bài 1. Tìm các giá trị thực của tham số m để dãy số  xn  :  m có giới hạn hữu hạn.
 xn 1  1  x 2 n  N
*

 n

Hướng dẫn giải


*) m  0  0  xn  m n  1 .

m 2mx
Xét hàm số: f ( x)  ta có f '( x)  2  f  x  nghịch biến trên  0; m  .
x 1
2
( x  1) 2

Suy ra ( x2 n ), ( x2 n 1 ) đơn điệu và bị chặn.

2017  x1  x3  x5  ... 
+ 0m  x1  x2 , x3   .
2016  x2  x4  x6  ... 
4m m
f ( f (1))   1, x2   1  x2 n  1 n  N * .
m 4
2
2017


a(1  b )  m
2

Giả sử lim x2 n  a, lim x2 n 1  b  a  1,  (I ) .



b (1  a 2
)  m

 a  b
 3 ( II )
  a  a  m

( I )   b  1 .
  a

( III )
1
 a   m
  a

Khi o  m  2 hệ (I) có nghiệm duy nhất   xn  có giới hạn hữu hạn.

2017
Khi 2  m  hệ (II) có nghiệm duy nhất lớn hơn 1 và hệ (III) có nghiệm thỏa mãn a  b . Do đó
2016
 lim x2 n  lim x2 n 1  ( xn ) không có giới hạn.

2017  x1  x3  x5  ...
  m  2017 2016  x1  x2 , x1  x3   .
2016  x2  x4  x6  ...
 lim x2 n  lim x2 n 1  ( xn ) không có giới hạn.

+ m  2017 2016  xn  2016 n  N *  l imxn  2016 .

 x1  x3  x5  ...
+ m  2017 2016  x1  x2 , x1  x3   .
 x2  x4  x6  ...

Trang51
 lim x2 n  lim x2 n 1  ( xn ) không có giới hạn.

*) m  0 tượng tự ta có 0  m  2 và m  2017 2016 .

xn3  6 xn  6
Bài 2. Cho số thực a, xét dãy số  xn n1 được xác định bởi x1  a, xn1  , n  1, 2,.... .
3xn2  9 xn  7
Tìm tất cả các giá trị của a để dãy số có giới hạn hữu hạn, tìm giới hạn đó?.

Hướng dẫn giải


Với a  1 thì xn  1, n  1 nên lim xn  1 .
n 

 xn1  1  xn1  2
3 3

Với a  1 thì xn  1  , xn  2  , n  2 .
3xn21  9 xn1  7 3xn21  9 xn1  7
3 3n1
x  2  xn 1  2  a2
Do đó n    ...   , n  1 .
xn  1  xn 1  1   a 1 
3n1 3n1
2  a  1   a  2
Từ đó, tính được xn  3n1 3n1
, n  1 ,.
 a  2   a  1

3
Kết luận + a    a  1  a  2  lim xn  2 .
2 n 

3
+ a    a  1  a  2  lim xn  1 .
2 n 

3 3 3
+ a    xn   , n  1  lim xn   . .
2 2 n  2

 1  an 1
an  bn  1  a
Bài 3. Cho hai dãy số dương  an n0 ,  bn n0 xác định bởi: a0  3, b0  2 và  n 1 .
a 2  1  b 2
 n n

Với mọi n  0,1, 2,... . Chứng minh rằng hai dãy trên hội tụ và tìm giới hạn của chúng.

Hướng dẫn giải


 1
Ta chứng minh bằng quy nạp an  tan , bn  , n  0,1, 2,... (*) . Thật vậy.
3.2 n

cos
3.2n
 
, vậy * đúng.
1
Với n  0 , ta có a0  3  tan  tan , b0  2 
3 3.2 0

cos
3.20
 
, vậy * đúng.
1 2 1
Với n  1 , ta có a1   tan  tan 1 , b1  
3 6 3.2 3 cos 
3.21
 1
Giả sử khẳng định đúng đến n  k , k  1 , tức là an  tan , bn  .
3.2 n

cos
3.2n
Trang52

. Thật vậy. Từ 1 ta có.
1
Ta chứng minh an1  tan , bn1 
3.2 n 1

cos
3.2n1
    
sin 1 2sin cos  sin 2  cos 2
1  an 1 3.2 n
3.2 n 1
3.2 n 1
3.2 n 1
3.2n 1 
 
1  an 1   
cos n cos 2 n 1
 sin 2
3.2 3.2 3.2n 1
  
2

 sin n 1  cos n 1 
  3.2 3.2 
      
 cos n 1  sin n 1  cos n 1  sin n 1 
 3.2 3.2  3.2 3.2 
  
sin  cos tan 1
3.2 n 1
3.2 n 1
3.2n 1 
  a  tan
   n 1
3.2n 1
cos n 1
 sin 1  tan
3.2 3.2n 1 3.2n 1

Khi đó từ  2  , suy ra bn21  an21  1  tan 2
1 1
1   bn1  .
3.2 n 1
 
cos2 cos
3.2n1 3.2n1
 1
Như vậy theo nguyên lý quy nạp thì an  tan , bn  , n  0,1, 2,... .
3.2 n

cos
3.2n
 1 1
Do đó lim an  lim tan  tan 0  0; lim bn  lim  1 .
n  n  3.2 n
n  n   cos 0
cos
3.2n
Kết luận: lim an  0; lim bn  1 .■.
n n

u1  2014
Bài 4. Cho dãy số (un ) xác định như sau :  . Tìm điều kiện của
un 1  un  (1  2a)un  a ; n  1, 2,...
2 2

a để dãy số (un ) có giới hạn hữu hạn khi n   và tính giới hạn đó.

Hướng dẫn giải

Ta có: un1  un  (un  a)2  0  un1  un ; n  1, 2,3,... .

* Suy ra dãy số (un ) tăng knn ; từ đó dãy số (un ) có giới hạn hữu hạn khi và chỉ khi dãy bị chặn trên.

Giả sử lim un  L ( L  ) , thì chuyển qua giới hạn hệ thức un1  un2  (1  2a)un  a2 ta có:
n 

L  L  (1  2a) L  a 2  L  a .
2

- Nếu có chỉ số k  *
mà uk  a thì un  a; n  k trái với kết quả lim un  L  a .
n 

Do đó: uk  a với mọi k  1, 2,... hay un2  (1  2a)un  a2  a, n  1, 2,3,... .

 a  1  u1  a  a  1  2014  a .

* Đảo lại: Nếu a  1  2014  a  a  1  u1  a .

Trang53
 (u1  a  1)(u1  a)  0  u12  (1  2a)u1  a2  a  0  u2  a .

và u1  u2  a  1  u2  a .

Bằng quy nạp ta chứng minh được a  1  un  a, n  1, 2,3,... (H/s trình bày ra).

Như vậy dãy (un ) tăng knn, bị chặn trên bới a , do đó dãy số (un ) có giới hạn hữu hạn.

Kết luận: Với điều kiện a 1  2014  a thì dãy số (un ) có giới hạn hữu hạn khi n   và lim un  a .
n 

 x1  a

Bài 5. Cho dãy số  xn  thỏa mãn  2 xn3 . Tìm a sao cho dãy số xác định và có
 n 1 3x 2  1 , n  1, 2,3,...
x 
 n

giới hạn hữu hạn.

Hướng dẫn giải

2 x3 3
Đặt f  x   ,x   . Ta có x1  a, xn1  f  xn  . Ta có.
3x  1
2
3

6x4  6x2 6 x 2  x 2  1
f ' x   .
 3x2  1  3x2  1
2 2

Bảng biến thiên.

3
Ta xây dựng dãy số như sau a0  , a0  f  a1  , a1  f  a2  , a2  f  a3  ,... .
3
Nhận thấy a1 , a3 ,..., a2 k 1 ,...  0; a0 , a2 ,..., a2 k ,...  0 .

 3   3
Dựa vào bảng biến thiên ta thấy a1    ;0  , a2  f 1  a1    0;  .
 3   3 

 a2  a0  f  a3   f  a1   a3  a1  f  a4   f  a2   a4  a2 .

3
Bằng quy nạp ta chứng minh được dãy  a2k  đơn điệu giảm, bị chặn bởi 0 và , dãy  a2 k 1  đơn điệu
3
3
tăng và bị chặn bởi  và 0. Từ đó tồn tại lim  a2 k  , lim  a2 k 1  .
3 k  k 

Ta có an  f  an1   f  f  an 2    lim an  f  f  lim an 2    l  f  f  l   .


3
 2l 3 
2 2 
3l  1   1
l    l  l 2    l 2  1 20l 4  15l 2  5   0 (*).
 5
2
 2l 3 
3 2  1
 3l  1 

2 x3 3  3   3
(do f  x   ,x   liên tục trên   ;0  ,  0;  và l  nlim an ).
3x  1
2
3  3   3 


Trang54
. Ta có f  f  an    an  an  2  an  0 nên * 
3 1 3 5
Xét 0  l   an  . Vậy l  .
3 5 3 5

5
Tương tự ta chứng minh được dãy  a2 k 1  đơn điệu tăng, hội tụ về  .
5
 5
 nÕu n ch½n
5 
+) Nếu a  thì x2   x1 , x3   x2 nên ta có dãy xn   5 .
5  5
 5 nÕu n lÎ

Dãy này không hội tụ.

 5
 nÕu n ch½n
5 
+) Nếu a   ta có dãy xn   5 .
5  5
 5 nÕu n lÎ

Dãy này không hội tụ.


+) Nếu tồn tại n sao cho a  an thì ta có.

3
x1  an  f  x1   f  an   x2  an1  f  x2   f  an1   x3  an2 ,..., xn1  a0  .
3
Khi đó không tồn tại xn  2 .

Vậy nếu a  an thì dãy không xác định.

5
+) Nếu 0  a  thì hai dãy con  x2 k  ,  x2 k 1  cùng hội tụ về 0 nên giới hạn của dãy là 0.
5
Nếu a  1 thì x2  f  a   a  x1 và hàm số đồng biến nên dãy đơn điệu giảm, bị chặn dưới bởi 1. Khi đó
dãy hội tụ về 1.

3
+) Nếu  a  1 thì x2  f  a   1 . Khi đó ta có thể khảo sát dãy từ x2 . Trường hợp này dãy đơn điệu
3
giảm và bị chặn dưới bởi 1 nên hội tụ về 1 .
+) Nếu a = 1 thì xn  1 n nên dãy hội tụ về 1 .

5 3 5 3
+) Nếu a ta có  lim a2 n và a0  nên tồn tại a2 k , a2 k  2 sao cho a2 k  2  a  a2 k (Thật
5 3 5 n 3
3
vậy, các số hạng của  a2k  không thể cùng nằm bên trái a do a0  , chúng cũng không thể cùng nằm
3
3 5
bên phải a do nếu thế thì a  a2 n   lim a2 n  ).
3 n  5

3
Vậy a   a2 k 2 ; a2 k   x2   a2 k ; a2 k 2  ,..., x2 k   a2 ; a0  , x2 k 2   a0 ;    x2 k 2  . Khi đó ta lại có
3
dãy đơn điệu giảm, bị chặn dưới bởi 1 nên hội tụ về 1.

Trang55
5 3
Vì f(x) là hàm lẻ nên trường hợp   a  0,  1  a   , a  1, a  1 ta khảo sát tương tự.
5 3
Kết luận: Điều kiện để dãy xác định và có giới hạn hữu hạn là.

3 5
a ;a ; a  an , n  1, 2,3,... .
3 5
n
Bài 6. Cho dãy số  an  xác định bởi 0  a1  1 và an1  an  , n  1 . Chứng minh rằng
an
lim  an  n   0 .
n 

Hướng dẫn giải


1
Áp dụng bất đẳng thức AM-GM ta có a2  a1   2 (do a1  1 ).
a1

Nhận xét: an  n, n  2 .

Ta sẽ chứng minh nhận xét này bằng phương pháp quy nap.
Thật vậy.
Với n  2 ta có a2  2 (đúng).

Giả sử ak  k .

k
Ta có ak 1  ak   k  1  ak2  k   k  1 ak .
ak

 ak2   k  1 ak  k  0 .

  ak  1 ak  k   0 (đúng).

Suy ra ak 1  k  1.

Như vậy an  n, n  2 (điều phải chứng minh).

n n
Mặt khác, an1   n  1  an    n  1  an  n   1 .
an an

an2   n  1 an  n  an  n  an  1
  (1).
an an

Áp dụng (1) ta có.

Trang56
  a2  2  a2  1
a3  3 
 a2
  a  3 a3  1
a4  4  3
 a3 .
 ...

  an  n  an  1
an 1   n  1 
 an

Suy ra  a3  3 a4  4  ...  an1   n  1  


 a2  2 a2 1 a3  3 a3 1 ...  an  n  an  1 .
a2 a3 ...an

 an1   n  1 
 a2  2 a2  1 a3  1 ...  an  1 .
a2 a3 ...an

 1  1  1
 an 1   n  1   a2  2  1   1   ... 1   .
 a2  a3   an 
n
 1
 an 1   n  1   a2  2   1   (2).
i2  ai 

n
an  1
1 a 1 an an n
Ta lại có 1   n 1   (do an  n   1 ).
an 1 an1 an1 an1 an
n
 1  a1 a2 an 1 a1
Suy ra  1  a   . ...  .
i2  i  a2 a3 an an

a1 a
Từ (2)  an1   n  1   a2  2  .   a2  2  . 1 (vì an  n ).
an n

a1
 0  an 1   n  1   a2  2  . .
n
a1 a
Mà lim  0  lim  a2  2  1  0 .
n  n n  n

Do đó lim  an 1   n  1   0 hay lim  an  n   0 .


n  n 

1 a 
Bài 7. Cho p  *
, a  0 và a1  0 . Xét dãy số (an ) được xác định bởi: an 1  ( p  1)an  p 1  ,
p an 
với mọi n  1. Chứng minh dãy số (an ) có giới hạn hữu hạn khi n  . Hãy tìm giới hạn đó.

Hướng dẫn giải


* Theo bất đẳng thức Côsi ta có:.

1  a  1 a
an 1  an  an  ...  an  p 1  p. p anp 1. p 1  a , với  n  1 . (1).
p

p an  p an
 p 1 

Trang57
1 a 
an 1  an  ( p  1)an  p 1   an
p an 
Do đó: .
a a anp  a
 n     0;  n  2 (2)
p p.anp 1 p.anp 1
p
Từ (1) và (2) ta có dãy số (an ) giảm và bị chặn dưới bởi a ;.

suy ra dãy số (an ) có giới hạn hữu hạn khi n  . .

Giả sử lim an  L ; ( L  a ).
p
n

1 a 
Chuyển qua giới hạn hệ thức an 1  ( p  1)an  p 1  .
p an 

1 a 
ta có phương trình L   ( p  1) L  p 1   pLp  ( p  1) Lp  a .
p L 

 Lp  a  L  a (thỏa mãn điều kiện).


p

Vậy lim an  a .
p
n 


1 
Bài 8. Cho trước số thực dương  và xét dãy số dương  xn  thỏa mãn xn1     1   1 với
xn
mọi n  *
. Chứng minh rằng dãy  xn  hội tụ và tìm giới hạn của nó.

Hướng dẫn giải


1
Xét hàm số f ( x)  x  , x  0 .
x

1  x 1  1 
1
Ta có f '( x)   x 1   ; f '( x )  0  x  x0    1
.
x2 x2
Ta có bảng biến thiên của hàm f(x):.

x 0 x0 +∞

f'(x) 0 +
+∞ +∞
f(x)

f(x0)
.
 1 
 
Suy ra f ( x)  f  x0     1
  1
 (  1)  1
.

1  1
Do đó xn1     1  1  xn1 
  .
xn xn1

Suy ra xn 1  xn hay  xn  là dãy giảm. Kết hợp với xn  0 với mọi n ta suy ra dãy  xn  hội tụ.

Trang58

1 
Đặt lim xn    0 . Chuyển qua giới hạn ta được     (  1)  1
   x0 .

1

Vậy lim xn    1 .

un  (0;1)
Bài 9. Tìm tất cả các hằng số c  0 sao cho mọi dãy số dãy số (un ) thỏa mãn  n  1
un 1 (1  un )  c
đều hội tụ. Với giá trị c tìm được hãy tính giới hạn của dãy (un ) .

Hướng dẫn giải


Ta xét các trường hợp sau.
1 c cun
+ Nếu c  , thì từ giả thiết, ta có un1    4cun ; n  1 .
4 1  un un (1  un )

1
Từ đây bằng quy nạp, ta suy ra un  (4c)n1 u1 . Do 4c  1 nên un   khi n   . Do đó, c 
4
không thỏa mãn.

1  1  1  4c 1  1  4c  a(1  b)  c
+ Nếu 0  c  , thì tồn tại a, b   ;  , a  b sao cho  . Thật vây, lấy
4  2 2  b(1  a)  c
 1  1  4c 1  1  4c 
a   ;  , đặt b  a  x ( x  0) , thì.
 2 2 
a (1  a )  c
a (1  b)  c  a (1  a  x)  c  x  .
a
Chú ý là b(1  a )  a (1  a )  c. Do đó, ta chỉ cần chọn x  0 như trên và b  a  x, thì được 2 bất đẳng
thức nêu trên.
Xét dãy số (un ) xác định bởi.

a khi n  2m
un   .
b khi n  2m  1
1
thì dãy (un ) thỏa mãn giả thiết nhưng không hội tụ. Thành thử, 0  c  cũng không thỏa mãn.
4
1 1 un
+ Nếu c  , thì un1    un . Suy ra dãy (un ) tăng và bị chặn. Do đó, (un ) hội tụ.
4 4(1  un ) 4un (1  un )

1 1 1
Đặt x  lim un , thì từ giả thiết ta có x(1  x)  hay x  . Vậy lim un  . .
4 2 2

Bài 10. Cho dãy số  un  xác định như sau: u1  2 , un1  un 2  un  1 , n  *


. Tìm giới hạn của dãy
1 1 1
 sn  với sn    ...  , n  *
.
u1 u2 un

Hướng dẫn giải

Trang59
Bằng phép quy nạp đơn giản ta thấy rằng: un  2 .

Xét tính đơn điệu của dãy  un  . Từ hệ thức un1  un 2  un  1 ta suy ra được
n  , un1  un   un  1  0 , vậy dãy số  un  tăng.
* 2

Tính tổng: Từ hệ thức truy hồi (1) ta suy ra được un 1  1  un  un  1 .

1 1 1 1 1 1 1
       * với n *
.
un 1  1 un  un  1 un  1 un un un  1 un1  1

Thay n bởi 1, 2, 3,., n vào (*) và cộng vế với vế các đẳng thức ta suy ra :.
1 1 1 1
  ...   1  .
u1 u2 un un1  1

Do dãy  un  là dãy tăng nên có hai khả năng sau xảy ra:.

1) Dãy  un  bị chặn trên. Theo tiêu chuẩn Weierstrass, nên  un  tăng và bị chặn trên nên nó có giới hạn.
Giả sử lim un  a  a  2 . Chuyển qua giới hạn hệ thức (1) khi n   ta có:
n

a  a 2  a  1  a 2  2a  1  0  a  1 , vô lý.

2) Dãy không bị chặn trên, do  un  tăng và không bị chặn trên nên


1
lim un    lim  un  1    lim  0.
n  n  n un

1 1 1  1
Vì thế từ (2) ta suy ra: lim    ...    lim 1    1 .
n  u un  n  un 
 1 u2

1 un3
Bài 11. Cho dãy số (un) thỏa mãn : u0  2016; un1  un  . Tính lim .
un2 n  n

Hướng dẫn giải


3
 1  3 1
(un 1 )   un  2   un3  3  3  6 .
3

 un  un un

3 1
Do un  0 n => (un1 )3  un3  3  3
 6  un3  3 , n .
un un

suy ra (un )3  u03  3n  20163  3n, n  (1).

Lại có.
3
 1  3 1
(un 1 )   un  2   un3  3  3  6
3

 un  un un
.
3 1 1 1
 un  3 
3
  un  3  
3

20163  3n  20163  3n 2 n  3n 2

Trang60
1 1
=> (un 1 )3  un3  3   n  .
n  3n 2

Suy ra.
n 1
1 n1 1 n
1 n 1
(un )3  u13  3(n  1)     2  u13  3n     2 .
k 1 k k 1 9k k 1 k k 1 9k

n
1 1 1 1 1
Do k
k 1
2
 1 
1.2 2.3
 ... 
(n  1)n
 2  2.
n
2
 n 1  n
1
và     n 2  2n (Bất đẳng thức Bunhiacopxki).
 k 1 k  k 1 k

2
suy ra (un )3  u13  3n   2n (2).
9
Từ (1) và (2) suy ra.
2
20163  3n  (un )3  u13  3n 
 2n , n 
9
.
20163 (u )3 u 3 2 2
 3  n  1 3  , n 
n n n 9n n

un3
Do đó lim  3.
n  n

Bài 12. Cho số thựca, xét dãy số  xn  xác định bởi:


x1  a, xn1  ln  3  cos xn  sin xn   2014, n  1, 2... Chứng minh rằng dãy số trên có giới hạn
hữu hạn khi n  . .

Hướng dẫn giải

Đặt f  x   ln  3  sin x  cos x   2014, x  .

cos x  sin x
 f ' x  .
3  sin x  cos x

   
 3 f '  x   2 cos  x    2 f '  x  sin  x  
 4  4
.
 9  f ' x   2  2  f '  x 
2
 f ' x 
2 2
 q, x 
7

Áp dụng định lí Lagrange cho hàm số f  x  liên tục và có đạo hàm trên , thì với mọi số thực x,y tồn tại
z sao cho:.

f  x   f  y   f '  z  x  y  q x  y  f  x   f  y   q x  y , x, y  .

Với m  n  m, n  *
 , ta có: xm  xn  f  xm1   f  xn1   q xm1  xn1  ...  q mn1 xmn1  x1 .

Mặt khác: 2014  xn  2014  ln 5, n  *


  xn  bị chặn.

Trang61
Do đó:   0, N  *
: q mn1 xmn1  x1   , m  n  N . .

Vậy  xn  là dãy Cauchy, nên dãy số đã cho hội tụ.

un 1  vn 1
Bài 13. Cho hai dãy số un  và vn  xác định như sau: u1  1, v1  2, và un  , vn  un vn 1 khi
2
n  2 . Chứng minh rằng hai dãy un  và vn  có giới hạn và tìm giới hạn đó.

Hướng dẫn giải


 1  a b
Ta có cos  suy ra u1  cos v1 mà an  n 1 n 1 , bn  anbn 1 khi n  2 .
3 2 3 2
 
u v
Suy ra u2  1 1  2 cos 2 3 , v2  u2 v1  2 cos 3 .
2 2 2
  
u v 
u3  2 2  2 cos 3 cos 2 3 , v3  u3v2  2 cos 3 cos .
2 2 4 2 3
bằng phương pháp quy nạp ta chứng minh được.
   
u v
un  n 1 n 1  2 cos 3 cos 32 cos 33 ....cos 2 3n 1 .
2 2 2 2 2
   
vn  un vn 1  2 cos 3 cos 32 cos 33 ....cos 3n 1 .
2 2 2 2
sin 2
Mặt khác cos   nên ta có.
2 sin 
 
 
sin sin 3 sin 2 n32
3 . 1 
un  2 2 .... 2  n  2 sin cot 3n 1 .
   2 3 2
2sin 3 2sin 32 2 sin 3n 1
2 2

2 2 2
 
 
sin sin 3 sin n3 2 sin
3 . 2 ....... 2 1 3 .
vn  2  n2
   2 
2sin 3 2sin 23 3
2sin n 1 sin 3n 1
2 2 2 2
Do đó.

Trang62
  
    3 
 1     cot n 1 
lim un  lim  n  2 sin cot 3n 1   2sin lim  n21 
n  n  2
 3 2  3 n  2 
   
 
 
.

 3 
2sin 2sin
 3 lim 2n 1  3 3 3
 n    
3 tan 3n 1 3
2

 
n
Bài 14. Với mỗi n  *
, đặt Qn  x    x  i 2 .
i 0

a) Chứng minh đa thức Qn  x  có duy nhất 1 nghiệm thực xn thuộc  0;1 .

b) Chứng minh tồn tại giới hạn của dãy  xn  .

Hướng dẫn giải


a) Ta có Qn  0   Qn 1  Qn  22   ...  Qn  n2   0 .

 
nên trong mỗi khoảng  0;1 , 1; 4  ,...,  n  1 ; n 2 có 1 nghiệm của phương trình Qn  x   0 .
2

Mặt khác, ta có det Qn  x   n nên đa thức Qn  x  có duy nhất 1 nghiệm xn thuộc khoảng  0;1 . .
1 1 1 
b) Ta có Qn  x   Qn  x     ...  .
 x x 1 x  n2 
2

Do Qn  x  có nghiệm không là nghiệm của Qn  x  nên nghiệm của phương trình Qn  x   0 là nghiệm
của phương trình:.
1 1 1
fn  x     ...  0.
x x 1 2
x  n2
1 1 1
Ta có: f n  x    2   ...   0.
x  x  1  x  n2 
2 2

Nên f n  x  nghịch biến trên  0;1 .


1 1 1
Lại có: f n  xn     ...  0.
xn xn  12
xn  n2
1 1 1 1 1
    ...   0.
xn   n  1 xn xn  1 xn  n xn   n  1
2 2 2 2

 f n1  xn   0  f n  xn   f n1  xn1   xn  xn1 .


Do đó dãy  xn  là dãy giảm.
Lại có xn   0;1 . Vậy dãy  xn  có giới hạn.

Bài 15. Cho x1  a, x2  b  a, b   và n.xn  2  (n  1).xn 1  xn  0 , n  1, 2,... Tìm lim xn .


n

Trang63
Hướng dẫn giải
xn 1  xn
Ta có xn  2  xn 1   .
n

(1) n (1) n
 xn  2  xn 1  ( x2  x1 )   .b  a  .
n! n!
n
(1)k n
(1) k
 xn  2  x1   .  b  a   x1   a  b    .b  a  .
k 1 k! k 0 k!
1 1
 lim xn  x1  a  b   2a  b  .
e e

Bài 16. Cho dãy  un  axác định bởi: u1  2; un1  un2  un  1, n  *


. Tìm M nhỏ nhất thỏa mãn
1 1 1
  ...   M , n  *
.
u1 u2 un

Hướng dẫn giải


Ta có u1  2  1 và un1  (un  1)2  un . Chứng minh bằng quy nạp ta được un  2, n  , n  2 (*).

Ta lại có: ui 1  ui2  ui  1  ui 1  1  ui (ui  1) .

1 1 1 1 1 1
      .
ui 1  1 ui  1 ui ui ui  1 ui 1  1
n
1 1 1 1 (*)
Do đó: 
i 1 ui
 
u1  1 un 1  1
 1 
un 1  1
 1, n  *
.

Suy ra M  1 .
Mặt khác, chứng minh bằng quy nạp ta được dãy (un ) tăng. Do đó nếu dãy có giới hạn hữu hạn L thì
L  2 . Vì phương trình L  L2  L  1 có duy nhất nghiệm là L  1 , bởi vậy dãy (un ) không có giới hạn
 n 1 
hữu hạn. Suy ra lim un    lim     1 (**).
 i 1 ui 

 n 1 n0
1
Với mọi a  1 thì từ lim     1 suy ra tồn tại n0 sao cho u  a . Do đó M  1  M  1 .
 i 1 ui  i 1 i

Bài 17. Cho 4028 số thực: a1 , a2 ,..., a2014 , b1 , b2 ,..., b2014 . Xét dãy số  xn  xác định như sau:.
2014
xn    ai .n  bi ,  n  1, 2,3,... .
i 1

2014
Biết dãy số lập thành một cấp số cộng, chứng minh rằng a i là số nguyên (với  a  là phần nguyên của
i 1

số thực a – số nguyên lớn nhất không vượt quá a ).


Hướng dẫn giải

Trang64
2014 2014
Đặt A   ai , B   bi . Gọi d là công sai của cấp số cộng  xn  , thì: n.d  xn 1  x1 .
i 1 i 1

Với mọi n  *
ta luôn có: ai .n  bi  1   ai .n  bi   ai .n  bi , i  1, 2,..., 2014 .

Cộng vế với vế của 2014 bất đẳng thức cùng chiều, ta được:.
A.n  B  2014  xn  A.n  B .

Thay n bởi n  1 và thay n bởi 1 , có:.

A  n  1  B  2014  xn1  A  n  1  B .

A  B  2014  x1  A  B   A  B   x1   A  B  2014 .

Cộng vế với vế của 2 bất đẳng thức cùng chiều nói trên thu được:.
A.n  2014  xn 1  x1  A.n  2014 .

 An
.  2014  n.d  An
.  2014 .
 d .n  A.n  2014 .

2014
 dA  .
n
2014
Vì lim  0 nên suy ra d  A . Mặt khác dãy  xn  gồm toàn số nguyên nên công sai d cũng là số
n
nguyên. Vậy A nguyên. (đpcm).

 1
 x1 

Cho dãy số  xn 
2
Bài 18. thỏa mãn:  2
. Chứng minh dãy số trên có giới hạn.
 x  x  n ; n  1
x


n 1 n
n2

Hướng dẫn giải


n  n  1
*) Ta chứng minh xn  n2  với mọi n  1 (1).
2
Thật vậy : n  1 đúng.
k  k  1
Giả sử (1) đúng với n  k  1 : xk  k 2  .
2

xk2
 xk 1   k  1  xk    k  1 .
2 2
2
k

2  k
x  k 2    k  1 .
xk 2
=
k

 k  1  k  k  1
  1   k  1 .
2

k 2  2

Trang65
3  k  1 k  k  1
2

  .
2 2

k  1  3  k  1   k  1 k  2 
  k  (đpcm).
2  2  2

*) Ta chứng minh  xn  có giới hạn.

NX:  xn  tăng và xn  0 với mọi n .

1 1 1 2
Ta có    .
xn xn 1 xn  n 2
n  n  1

1 1  1
   2 1    2 .
x1 xn  n
1
 xn  với mọi n  1.
2 2

Vậy  xn  có giới hạn.

Bài 19. Cho dãy số  an  tăng, an  0n  1, 2,3,.... và   0 . Xét dãy số  xn  xác định bởi
n
ai 1  ai
xn   . Chứng minh rằng tồn tại lim xn .
i 1 ai 1ai n 

Hướng dẫn giải

Dễ dàng thấy rằng dãy  xn  tăng ngặt.

Trường hợp 1. Nếu   1 .


ai 1  ai 1 1 1 1 1

   1
     xn   .
ai 1ai ai ai 1ai ai ai 1 a1

vậy dãy  xn  bị chặn trên do đó tồn tại lim xn


n  .
Trường hợp 2. Nếu 0    1 .

ai 1  ai 1  1 1 
      * thật vậy *   ai11  ai 1  ai   ai1  ai .

ai 1ai   ai ai 1 

ai1  ai
  ai 1 ** .
 1

ai 1  ai

Ta chứng minh (**).

Xét hàm số f  x   x Trên đoạn  ai ; ai 1  .

Hàm số thoả mãn điều kiện của định lí Lagrăng nên tồn tại số c   ai ; ai 1  thoả mãn
ai1  ai a  a a  a
f c    c 1  i 1 i   ai11  i 1 i (đpcm).
ai 1  ai ai 1  ai ai 1  ai
Trang66
1
Từ đó ta có  xn   dãy  xn  bị chặn trên do đó tồn tại lim xn
 a1 n  .
n
1
 1 n  1, 2,3,... . Đặt S n  
a1a2 ...an
Bài 20. Cho dãy số xác định bởi a0  1; a1  1; an 1  .
a n  k 1 ak 1a k 
2  
  2

Chứng minh tồn tại lim Sn ( trong đó  x  là phần nguyên của x ).


n

Hướng dẫn giải


1 1 a 1 1 1
Ta có   k 1   .
ak 1a  k  a a1a2 ...ak a1a2 ...ak 1 a1a2 ...ak a1a2 ...ak 1
k 1
2
  ak 1  1
n
 1 1  1 1
Suy ra Sn       .
k 1  a1a2 ...ak a1a2 ...ak 1  a1 a1a2 ..an 1

Chứng minh lim  a1a2 ...an 1    .


n 

Ta có : an  1 n  2 .

n
 2   n  an1  an  1 suy ra dãy đã cho là tăng.

Như vậy an  an1  1  ...  a1  n  1 .

1
Vậy lim  a1a2 ...an 1    , suy ra lim Sn  .
n  n  a1

u1  3, v1  2

Bài 21. Cho dãy số  un  ;  vn  được xác định như sau un 1  un2  2vn2  n  N 
v  2u v
 n 1 n n
.

Tìm các giới hạn sau: lim 2n vn và lim 2n u1.u2 ...un


x  x 
.
Hướng dẫn giải

 
2
Ta có: n  N : un1  2.vn1  un2  2vn2  2 2.unvn  un  2.vn (1).

 
2
Áp dụng (1) ta suy ra: un  2.vn  un 1  2.vn 1 .

     
2n1 2n1 2n
Theo quy nạp ta có: un  2.vn  u1  2.v1  3 2 2  2 1 (2).

 
2n
Lập luận tương tự ta cũng có: un  2.vn  2 1 (3).

Trang67
 1
   
2n 2n 
u 
 n 2 2  1  2  1 
  
Từ (2) và (3) ta suy ra:  .
v  1  2  1 2  2  1 2 
   
n n

 n 2 2  

1
    
 
2n 2n 2n
Lại có: un  2 1  2 1  2 1 , từ đó suy ra: 2n un  2  1 .
2  

   
2n 2n
2 1 2 1
1 
    
2n 2n
2n
Tương tự ta có : vn  2 1 2 1   2n vn  .
2 2   8 8

Mặt khác ta có: vn  un . Do đó ta có dãy bất đẳng thức sau:.

 
1 2n
2 1
 
 1  2 2n n

2 1     2n vn  2n un  2  1 .
8 8

Như vậy theo định lí kẹp ta suy ra lim 2n un  lim 2n vn  2  1 .


n n

vn1
Hơn nữa theo đề bài ta có: vn1  2un vn  un  .
2vn

v2 v3 vn1 vn1 vn1


Suy ra: u1.u2 ...un  . ...   .
2v1 2v2 2vn 2n v1 2n1

vn 1 1
Vậy lim 2 u1.u2 ...un  lim 2n  lim 2n vn 1 .lim 2n n 1 .
n

n  n  n 1  
2 n n 2

1 1
 lim 2n 2un vn .lim 2n  lim 2 2.lim 2n un .lim 2n vn .lim 2n n1 .
n

n  n  n 1 n  n  n  n 
2 2

 1.  2 1 .  
2  1 .1  3  2 2 .

Tóm lại ta có: lim 2n vn  2  1 và lim 2n u1.u2 ...un  3  2 2 .


n  n

n
Bài 22. Cho dãy số  an  xác định bởi 0  a1  1 và an1  an  , n  1 . Chứng minh rằng
an
lim  an  n   0 .
n 

Hướng dẫn giải


1
Áp dụng bất đẳng thức AM-GM ta có a2  a1   2 (do a1  1 ).
a1

Nhận xét: an  n, n  2 .

Ta sẽ chứng minh nhận xét này bằng phương pháp quy nap.
Thật vậy.

Trang68
Với n  2 ta có a2  2 (đúng).

Giả sử ak  k .

k
Ta có ak 1  ak   k  1  ak2  k   k  1 ak .
ak

 ak2   k  1 ak  k  0 .

  ak  1 ak  k   0 (đúng).

Suy ra ak 1  k  1.

Như vậy an  n, n  2 (điều phải chứng minh).

n n
Mặt khác, an1   n  1  an    n  1  an  n   1 .
an an

an2   n  1 an  n  an  n  an  1
  (1).
an an

Áp dụng (1) ta có.

  a2  2  a2  1
a3  3 
 a2
  a  3 a3  1
a4  4  3
 a3 .
 ...

  an  n  an  1
an 1   n  1 
 an

Suy ra  a3  3 a4  4  ...  an1   n  1  


 a2  2 a2 1 a3  3 a3 1 ...  an  n  an  1 .
a2 a3 ...an

 an1   n  1 
 a2  2 a2  1 a3  1 ...  an  1 .
a2 a3 ...an

 1  1  1
 an 1   n  1   a2  2  1   1   ... 1   .
 a2  a3   an 
n
 1
 an 1   n  1   a2  2   1   (2).
i2  ai 

n
an  1
1 a 1 an an n
Ta lại có 1   n 1   (do an  n   1 ).
an 1 an1 an1 an1 an
n
 1  a1 a2 an 1 a1
Suy ra  1  a   . ...  .
i2  i  a2 a3 an an

Trang69
a1 a
Từ (2)  an1   n  1   a2  2  .   a2  2  . 1 (vì an  n ).
an n

a1
 0  an 1   n  1   a2  2  . .
n
a1 a
Mà lim  0  lim  a2  2  1  0 .
n  n n  n

Do đó lim  an 1   n  1   0 hay lim  an  n   0 .


n  n 


1 
Bài 23. Cho trước số thực dương  và xét dãy số dương  xn  thỏa mãn xn1     1   1 với
xn
mọi n  . Chứng minh rằng dãy  xn  hội tụ và tìm giới hạn của nó.
*

Hướng dẫn giải


1
Xét hàm số f ( x)  x  , x  0 .
x

1  x 1  1 
1
Ta có f ( x)   x  1
 2  ; f ( x)  0  x  x0   1 .

x x2

Ta có bảng biến thiên của hàm f  x  :.

x 0 x0 +∞

f'(x) 0 +
+∞ +∞
f(x)

f(x0)
.
 1 
 
Suy ra f ( x)  f  x0     1
   1  (  1)  1
.

1  1
Do đó xn1     1   1  xn1  .
xn xn1

Suy ra xn 1  xn hay  xn  là dãy giảm. Kết hợp với xn  0 với mọi n ta suy ra dãy  xn  hội tụ.

1 
Đặt lim xn    0 . Chuyển qua giới hạn ta được     (  1)  1
   x0 .

1

Vậy lim xn    1
.

u1 , u2  (0;1)

Bài 24. Cho dãy số thực  un  thỏa mãn  1 4 . Chứng minh rằng dãy (un ) có
un  2  un31  3 un , n  1
 5 5
giới hạn hữu hạn, tìm giới hạn đó.

Hướng dẫn giải


Trang70
 x1  min u1 , u2 

Xét dãy ( xn ) :  1 3 4 .
 xn 1  xn  3 xn
 5 5
Ta thấy xn  (0;1) .

1 4 x3  3 xn  3 xn  3 xn  3 xn 5 133
Ta có xn1  xn3  3 xn  n  xn  xn .
5 5 5
Vậy dãy  xn  tăng, bị chặn trên nên hội tụ, lim xn  a (0  a  1) .

1 4
Chuyển qua giới hạn ta được: a  a 3  3 a  a  1 .
3 5
Ta sẽ chứng minh xn  u2 n 1 ; u2 n  1 (*) bằng quy nạp theo n.

Ta có x1  u1 ; u2  1 . Giả sử xn  u2 n 1 ; u2 n  1 .

1 3 43 1 4
Suy ra xn 1  xn  xn  u23n  3 u2 n 1  u2 n 1  1 .
5 5 5 5
1 3 43 1 4 1 4
xn 1  xn  xn  xn31  3 xn  u23n 1  3 u2 n  u2 n  2  1 .
5 5 5 5 5 5
Vậy (*) đúng với mọi n nguyên dương. Từ đó suy ra lim un  1 .

 x1  2007

Bài 25. Cho dãy số thực  xn  xác định bởi:  xn . Chứng minh dãy số ( xn ) có

xn 1  3  n  1
 xn2 1
giới hạn và tìm giới hạn đó.

Hướng dẫn giải

Dễ dàng quy nạp xn  3 .

xn 1
Ta có: xn 1  3  = 3  1  3  2 n  1 .
x 1
2
n
x 1
2
n

Vậy xn  2007 với mọi n nên dãy bị chặn.

x 1 1
Xét f  x   3   f  x    f  x  khi x  3 .
x2 1 x  1
3
2 2 2

Ta có:.

x x2
f  x  x  x  3   ( x  3) 2 
x2 1 x2 1 .

 ( x 2  3x) 2  2( x 2  3 x)  3  0

Trang71
 x 2  3x  1 ( L)

 x  3x  3
2
.
3  15
x a
2
Áp dụng định lý Lagrang có:.
n
1  1 
xn 1  a  f ( xn )  f (a )  f '( n ) xn  a  xn  a  ...    x1  a 
n 
0 Do đó
2 2 2 2
3  15
lim xn  a  .
2
u1  e un21
Bài 26. Cho dãy số  un  xác định bởi:  . Tìm lim 2 2 2 .
un 1  un  2, n 
2 * n u .u ...u
1 2 n

Hướng dẫn giải


1
Vì u1  e  2 nên đặt u1  a  , a > 1.
a
2
 1 1
Ta có u2  u12  2   a    2  a 2  2 .
 a a

1
Bằng quy nạp, ta có thể chứng minh được un1  a 2  , n 
n

n .
a2
Xét.
1 1
n n
 i1 1   1   1  n  2i1 1   1   2n 1 

i 1
ui    a 2  2i1
i 1  a

 
 
a   
a  
a     a  2i1     a    a  2n 
a  i 1  a   a  a 
2
 1  2n 1 
 a   a  2n  2 2
.
a   lim un 1   a  1    a  1   4  e 2  4
2 2
 2 n21 2  
u a 
   
 a  a
2 n  u 2 .u 2 ...u 2
u1 .u2 ...un  2n 1 
 2n 
1 2 n
a
 a 

Bài 1. Cho dãy số  xn  xác định bởi.

 x1  a

 xn2  7 .
 n 1 2  x  3 , n  1, 2,3,...
x 
 n

Chứng minh rằng dãy số có giới hạn hữu hạn. Tính giới hạn đó.
Hướng dẫn giải
Theo Côsy thì.

1
xn   xn  3 
16   x  1 xn  7   0 .
 6   1; xn 1  xn   n
2 xn  3  2  xn  3

dãy giảm, bị chặn bởi 1, vậy dãy có giới hạn.

Trang72
Từ lim xn  a  a  1 .

 x1  1

Bài 27. Cho dãy số  xn  , xác định bởi:  2014 . Chứng minh rằng dãy số  xn  có
 x   1  , n  1, 2,3...
1  xn
n 1

giới hạn hữu hạn và tìm giới hạn đó.

Hướng dẫn giải


2014
Xét hàm số f ( x)  1  trên  0;   . Ta thấy f ( x ) liên tục và nghịch biến trên  0;   (Vì
1 x
2014
f '( x)   0 ). Do đó 1  f ( x)  2015 .
1  x 
2

2014
Ta có xn1  1   f ( xn ) với mọi n  dãy  xn  bị chặn.
1  xn

Mặt khác, ta có x1  x3  f ( x1 )  f ( x3 )  x2  x4  f ( x2 )  f ( x4 )  x3  x5  ... .Suy ra dãy  x2 n 1  là


dãy đơn điệu tăng và bị chặn, còn dãy  x2n  là dãy đơn điệu giảm và bị chặn, nên các dãy  x2 n 1  ,  x2n 
có giới hạn hữu hạn.
Giả sử lim x2 n 1  a và lim x2 n  b , ( a, b  1 ).

Từ x2 n1  f ( x2 n )  lim x2 n1  lim f ( x2 n )  b  f (a) .

x2 n  2  f ( x2 n 1 )  lim x2 n  2  lim f ( x2 n 1 )  a  f (b) .

 2014
b  1  1  a
Vậy ta có hệ   a  b  2015 .
a  1  2014
 1 b

Vậy lim xn = 2015 .

 x1  2,1

Bài 28. Cho dãy số  xn  được xác định bởi  xn  2  xn2  8 xn  4 với mỗi số
 xn1  * , n  1, 2,...
 2
n
1
nguyên dương n, đặt yn   2 . Tìm lim yn .
i 1 xi  4

Hướng dẫn giải


Ta có kết quả sau: với số thực a  2 bất kì, ta có.

a  2  a 2  8a  4 a  2  a 2  4a  4 a  2   a  2 
  a.
2 2 2
Do đó 2,1  x1  x2  ... Suy ra dãy  xn  là dãy tăng, giả sử bị chặn trên tức là có giới hạn lim xn  L  2 .

Chuyển qua giới hạn điều kiện (*) ta có phương trình.

Trang73
x  2  x2  8x  4
x  x 2  4   x  3 x  2  .
2
phương trình này không có nghiệm hữu hạn lớn hơn 2.

Suy ra dãy  xn  tăng và không bị chặn trên nên lim xn   .

xn  2  xn2  8xn  4
Ta có xn1   2 xn1  xn  2  xn2  8xn  4 .
2

  2 xn1  xn  2   xn2  8xn  4  xn22  4   xn  3 xn  2  .


2

1 x  3 xn  2  1 1 1
  2n  2   2 .
xn  2 xn1  4 xn1  4 xn1  2 xn1  4

1 1 1
   .
x  4 xn  2 xn1  2
2
n 1

n
1 1 1 1
Suy ra yn      10  .
i 1 x  4 x1  2 xn 1  2
2
i xn 1  2

Vậy lim yn  10 .

 x0  a

Bài 29. Dãy số thực  xn  n  
được xác định bởi:   n   . Tìm tất cả các giá trị

 n 1
x  2 xn
2
 1
của a để xn  0 với mọi số tự nhiên n.

Hướng dẫn giải


Giả sử xn  0 với n  .

2
Từ xn2  2 xn21 1  0 có   xn1  0 .
2

2 2  2 2 1
Lại từ   2 xn2  1  0 có   xn   1  xn   , n  .
2 2 2 4
1 3 1
Suy ra xn   và xn   1, n  .
2 4 2

1 1 1 1 1 3 1
Từ đó xn1   2 xn2  1   2 xn2   2 xn  . xn   xn  , n  .
2 2 4 2 2 2 2
Áp dụng liên tiếp bất đẳng thức này, ta có:.
2 n n
1 1 2 1 2 1 2 1 2
a  x0   x1     x2   ...    xn     , n  .
2 2 3 2 3 2 3 2  3
n
2 1 1
Mà lim    0 nên phải có a   0  a   .
 
n 3 2 2

Trang74
1 1
Thử lại với a   thì xn    0, n .
2 2
1
Vậy a   là giá trị duy nhất cần tìm.
2

 x1  2014
Bài 30. Cho dãy số thực (xn) xác định bởi:  .
 xn1  3 6 xn  6sin xn , n 
*

Hướng dẫn giải

x3
Sử dụng bất đẳng thức x   sin x  x, x  0 .
6
Xét hàm số f  x   3 6 x  6sin x , x  0 .

6 1  cos x 
Ta có: f '  x    0, x  0  f(x) luôn đồng biến với mọi x > 0.
 6x  6sin x 
2
33

Do đó: f  x   f  0   0 x  0 . mà x2  f  x1   0 vì x1  2014  0. .

Vậy ta có xn 1  f  xn   0, n  N * .

6 xn  6sin xn  xn3
Mặt khác: xn1  xn  6 xn  6sin xn  xn 
3 .
 6 xn  6sin xn 
2
3  xn 3 6 xn  6sin xn  xn2

x3
Vì x   sin x  x, x  0  6 x  x3 – 6sinx  0, x  0 .
6
 6 xn – 6sinxn  xn3  0 do xn  0  xn1 – xn  0 .

  xn  là dãy giảm và bị chặn dưới bởi 0 nên tồn tại giới hạn hữu hạn.

Giả sử limxn  x( x  0) , ta có phương trình:.

x  3 6x  6sin x  x3  6x  6sin x  0 .
Xét hàm số g  x   x3  6x  6sin x .
g '  x   3x 2 – 6  6cosx .

g’’  x   6 x – 6sinx  0x  0 .

 g’  x   g’  0   0 . Do đó g  x  luôn đồng biến và liên tục với mọi x  0 .

 phương trình g  x   0 có nghiệm duy nhất x  0 .

Vậy limxn  0 .

Trang75
 1  an 1
an  bn  1  a
Bài 31. Cho hai dãy số dương  an n0 ,  bn n0 xác định bởi: a0  3, b0  2 và  n 1 .
a 2  1  b 2
 n n

Với mọi n  0,1, 2,... . Chứng minh rằng hai dãy trên hội tụ và tìm giới hạn của chúng.

Hướng dẫn giải


 1
Ta chứng minh bằng quy nạp an  tan , bn  , n  0,1, 2,... (*) . Thật vậy.
3.2 n

cos
3.2n
  1
Với n  0 , ta có a0  3  tan  tan , b0  2  , vậy * đúng.
3 3.2 0

cos
3.20
1   2 1
Với n  1 , ta có a1   tan  tan 1 , b1   , vậy * đúng.
3 6 3.2 3 cos 
3.21
 1
Giả sử khẳng định đúng đến n  k , k  1 , tức là an  tan , bn  .
3.2 n

cos
3.2n
 1
Ta chứng minh an1  tan , bn1  . Thật vậy. Từ 1 ta có.
3.2 n 1

cos
3.2n1
    
sin n  1 2sin n 1 cos n 1  sin 2  cos 2
1  an 1 3.2 3.2 3.2 3.2 n 1
3.2n 1 
 
1  an 1   
cos n cos 2 n 1
 sin 2
3.2 3.2 3.2n 1
  
2
   
 sin n 1  cos n 1  sin n 1  cos n 1 tan n 1  1
 3.2 3.2 

     
3.2

3.2 

3.2

Khi đó từ  2 ,
  
 cos n 1  sin n 1  cos n 1  sin n 1  cos
3.2n 1
sin
3.2n 1
1 tan
3.2n 1
 3.2 3.2  3.2 3.2 

a n 1  tan
3.2n 1
 1 1
suy ra bn21  an21  1  tan 2 1   bn1  .
3.2 n 1
 
cos2 cos
3.2n1 3.2n1
 1
Như vậy theo nguyên lý quy nạp thì an  tan , bn  , n  0,1, 2,... .
3.2 n

cos
3.2n
 1 1
Do đó lim an  lim tan  tan 0  0; lim bn  lim  1 .
n  n  3.2 n
n  n   cos 0
cos
3.2n
Kết luận: lim an  0; lim bn  1 .■.
n n

Trang76
u1  2014
Bài 32. Cho dãy số (un ) xác định như sau:.  . Tìm điều kiện của
un 1  un  (1  2a)un  a ; n  1, 2,...
2 2

a để dãy số (un ) có giới hạn hữu hạn khi n   và tính giới hạn đó.

Hướng dẫn giải


Ta có: un1  un  (un  a)2  0  un1  un ; n  1, 2,3,... .

* Suy ra dãy số (un ) tăng knn; từ đó dãy số (un ) có giới hạn hữu hạn khi và chỉ khi dãy bị chặn trên.

Giả sử lim un  L ( L  ) , thì chuyển qua giới hạn hệ thức un1  un2  (1  2a)un  a2 ta có:
n 

L  L2  (1  2a) L  a 2  L  a .

- Nếu có chỉ số k  *
mà uk  a thì un  a; n  k trái với kết quả lim un  L  a .
n 

Do đó: uk  a với mọi k  1, 2,... hay un2  (1  2a)un  a2  a, n  1, 2,3,... .

 a  1  u1  a  a  1  2014  a .

* Đảo lại: Nếu a  1  2014  a  a  1  u1  a .

 (u1  a  1)(u1  a)  0  u12  (1  2a)u1  a2  a  0  u2  a .

và u1  u2  a  1  u2  a .

Bằng quy nạp ta chứng minh được a  1  un  a, n  1, 2,3,... .

Như vậy dãy (un ) tăng knn, bị chặn trên bới a , do đó dãy số (un ) có giới hạn hữu hạn.

Kết luận: Với điều kiện a 1  2014  a thì dãy số (un ) có giới hạn hữu hạn khi n   và lim un  a .
n 

u1  1

Bài 33. Cho dãy số (un ) xác định bởi công thức truy hồi  1 . Chứng minh
u n 1  u n   2, n  *

 un
rằng dãy (un ) có giới hạn hữu hạn và tính giới hạn đó.

Hướng dẫn giải


1 1 1
Đặt f ( x)  x   2; g ( x)  f ( f ( x))  x    2 2 . Khi đó.
x x x 1  2
x
 2 2
2  x    x  1
 2  1 1
g '( x)  2
 0  g ( x)  g ( )  0  f ( f ( x))  x, x  ( ;1) (*). .
4 1  2 2
x x  2
 x 
1
Mặt khác f '( x)  0, x  ( ;1) nên.
2

Trang77
1 1 1 1 1
f ( x)  f ( )  f ( f ( x))  f ( )  , x  ( ;1) (**). .
2 2 2 2 2
1 1
Từ (*) và (**) suy ra:  f ( f ( x))  x, x  ( ;1). .
2 2
1 1
Vậy: 1  u1  u3   1  u1  u3  u5  ,... Do đó (u2 n 1 ) là đơn điệu giảm và bị chặn dưới nên tồn
2 2
1
tại lim u2 n 1  ..
n  2

 1 
Vì f ( x ) liên tục trên  ;1 nên u2 n  f (u2 n 1 )  lim u2 n  f lim u2 n 1 
 2  n  n 

1
2
.. 
Vậy dãy (un ) được phân tích thành hai dãy con hội tụ tới cùng một giới hạn. Do đó dãy (un ) có giới hạn
1
bằng ..
2

u1  2
 n
uk
Bài 34. Cho dãy số  un  xác định 
 
1
 2
   
. Tính lim
n 
 u  1
.


u n 1 u n
2014
u n u n , n 1 k  1 k 1

Hướng dẫn giải


un  un  1
Theo giả thiết ta có: un 1   un mà u1  2 suy ra.
2014

2  u1  u2  u3  ....... do đó dãy  un  là dãy tăng.

Giả sử dãy  un  bị chặn trên suy ra lim un  L với  L  2  khi đó.


n 

un2  2013un L2  2012L L  0


lim un1  lim L  .
n 2014 2014 L  1

Vô lý do L  2 . Suy ra dãy  un  không bị chặn trên do đó.

1
lim un    lim 0.
n  n  un

Ta có.

un2  2013un
un 1   un  un  1  2014  un 1  un 
2014
.
un  1 1 
  2014   
un 1  1  un  1 un 1  1 

 1 1 
 Sn  2014     lim Sn  2014 .
 u1  1 un 1  1  x 

Trang78
 x1  2014
Bài 35. Cho dãy số thực  xn  xác định bởi:  . Tính lim xn ? .
 xn1  3 6 xn  6sin xn , n 
*

Hướng dẫn giải

x3
Sử dụng bất đẳng thức x   sin x  x, x  0 .
6
Xét hàm số f  x   3 6 x  6sin x , x  0 .

6 1  cos x 
Ta có: f '  x    0, x  0  f(x) luôn đồng biến với mọi x > 0.
 6x  6sin x 
2
33

Do đó: f  x   f  0   0x  0 . mà x2  f  x1   0 vì x1  2014  0. .

Vậy ta có xn1  f  xn   0, n  N *. .

6 xn  6sin xn  xn3
Mặt khác: xn1  xn  6 xn  6sin xn  xn 
3 .
 6 xn  6sin xn 
2
3  xn 3 6 xn  6sin xn  xn2

x3
Vì x   sin x  x, x  0  6 x  x3 – 6sinx  0 . x  0 .
6
 6 xn – 6sinxn  xn3  0 do xn  0  xn1 – xn  0 .

  xn  là dãy giảm và bị chặn dưới bởi 0 nên tồn tại giới hạn hữu hạn.

Giả sử limxn  x( x  0) , ta có phương trình:.

x  3 6x  6sin x  x3  6x  6sin x  0 .
Xét hàm số g  x   x3  6x  6sin x .
g '  x   3x 2 – 6  6cosx .

g   x   6 x – 6sinx  0," x  0 .

 g   x   g   0   0 . Do đó g  x  luôn đồng biến và liên tục với mọi x  0  phương trình g  x   0 có


nghiệm duy nhất x  0 .
Vậy limxn  0 .

Trang79

You might also like